Download as pdf or txt
Download as pdf or txt
You are on page 1of 134

45 NOTES TO PG-IMAGES

—Dr. Ankit Yadavendra, M.B.B.S., Dr.V.M.G.M.C., Solapur

16-MEDICINE, PSYCHIATRY

Elfin facies-William
synd

Hatchet facies-myotonic dystrophy

Hutchinson facies-peritonitis
Alport synd-EM-basket weave app

Berry aneurysm

normal JVP

normal pulse-S1→ Anacrotic notch(complete


open AV)→ percussion wave→ tidal wave→
dicrotic notch(A2)

types of pulse
Battle sign-#middle cranial fossa

RAcoon sign-subgaleal hge,


infraorbital ecchymosis, #Ant cran
fossa

Koh block design test

Bender Gestalt test(MC screen test organic dysfn)

Bhatia battery test

clock drawing test


Roschach inkblot test-10card, ambiguos stimul,
personality struct assessm,underlying thought
abnorm, 8,9,10-colour card

Henry Murray&Christiana Morgan thematic


apperception test-20card, semiambiguous
stimuli, personality dynamic assessm,
interpersonal conflict

rTMS(repetitive Transcranial Magnetic


Stimulation)=1.5-3tesla-

depression, OCD, refractory hallucination,


schizophrenia, PTSD, migrainous headache,
bradykinesia

VNS(Vagus Nerve Stimulation)

ECT-u/l, b/l
Oto Veraguth fold-depression

inverted omega-depression

-(N.B.-THESE NOTES ARE ONLY FOR THE PURPOSE OF


GUIDANCE AND HELP TO PG ASPIRANTS, NOT FOR
COMMERCIAL OR OTHER PURPOSE. IMAGES AND REFERENCE
HAVE BEEN TAKEN FROM INTERNET AND VARIOUS STANDARD
TEXTBOOKS. FOR ANY FEEDBACK/QUERY PLEASE CONTACT-
ankit.yadavendra@facebook.com or
dr.ankityadavendra@gmail.com )

for theory please refer-

35- MEDICINE https:/ / db.tt/ wNrnKADG ,

36- SIGN/ APPEARANCE, TEST, RULE https:/ / db.tt/ pw2oct21 ,

33- PSYCHIATRY https:/ / db.tt/ pvaGPwnE ,


Scanned by CamScanner
Scanned by CamScanner
Scanned by CamScanner
Scanned by CamScanner
Scanned by CamScanner
Scanned by CamScanner
Scanned by CamScanner
Scanned by CamScanner
Scanned by CamScanner
Scanned by CamScanner
Scanned by CamScanner
TOPI C 1: SCHI ZOPHRENI A PSYCHI ATRY 1

PSYCHI ATRY Generally one or ot her of the conflicting sides is usually


repressed.
TOPI C 1 : SCHI ZOPHRENI A
• (Thus, for example - in psychoanalysis of a patient,
1. The primary disturbance in Schizophrenia I s – ‘love’ for his fat her might be quit e consciously
A. Hallucination experienced and openly expressed – while his ‘hate’
B. Illusion for the same object might be heavily repressed and only
C. Psychomotor retardation indirectly expressed, and thus only revealed in analysis).
D. Formal thought disorder
D 4. Most common t ype of Schizophrenia is -
.........( AI I MS PGMEE - DEC 199 4) A. Simple
B. Hebephrenic
Apathy and absence of social interest characterize all types, C. Catatonic
especially the simple group who exhibit few additional D. Paranoid
symptoms. D
.........( AI I MS PGMEE - Dec - 199 5)
Silly grinning and talking to one-self distinguish the
hebephrenic. Most common type of Schizophrenia — Paranoid
• Schizophrenia with best prognosis — Catatonic
Delusions and hallucinations occur in most hebephrenic, • Schizophrenia with w orst prognosis — Hebephrenic
catatonic and paranoid patients. The latter often give • Schizophrenia associated with amphetamine —Paranoid
evidence of these symptoms by conversing or arguing
with imaginary individuals. • CATATONI C SCHI ZOPHRENI A. Catatonic schizophrenia
is a form of t hought disorder w ith prominent mot or
2. 18 yrs. old hears voices discussing him in third person symptoms and abnormalities. These symptoms include:
has : • Catalepsy, or motionlessness maintained over a long
A. Schizophrenia period of time.
B. Depression
C. Mania
D. Phobia
A
.........( AI I MS PGMEE - MAY 199 5)

• 3rd Person hallucination is Characteristics of Schizophrenia.

3. Ambivalence is most commonly seen in


A. Schizophrenia
B. Hysteria
C. Mania
D. Obsessive compulsive disorder
A • Ca t a t oni c ex ci t e m e nt , marked by agitation and
.........( AI I MS PGMEE - SEP 199 6) seemingly pointless movement.
• Catat onic st upor , with markedly slowed motor activity,
• Ambivalence is severe inabilit y t o decide for or often to the point of immobility and seeming unawareness
against of the environment.
• Am b i v a l e n ce i s a st a t e of h a v i n g e m ot i on s, • Catatonic rigidit y , in which the person assumes a rigid
t hought s or act ions in cont radict ion w it h each position and holds it against all efforts to move him or her.
ot her, w hen t hey are related t o an object , idea or • Catatonic posturing, in which the person assumes a bizarre
person ( for example, feeling both love and hatred or inappropriate posture and maintains it over a long period
for someone or somet hing) . of time.
• Found in : • Akinesia, or absence of physical movement.
• Schizophrenia (one of 4A’s of Blevler)
• Obsessive compulsive Neurosis

• it would not usually be expected that the person


embodying this ‘ambivalence’ would actually feel both
of the tw o cont radictory emot ions as such: except
in obsessional neurosis, which sees both sides being
more or less ‘balanced’ in consciousness

HELP LI NE NO. 9 39 1 56 7 70 7
TOPI C 1: SCHI ZOPHRENI A PSYCHI ATRY 2

• Waxy flexibilit y, in which the limb or other body part of This type of schizophrenia is also known as hebephrenia, and
a catatonic person can be moved into another position is named after the Greek goddess of youth, Hebe, in
that is then maintained. The body part feels to an observer reference to the typical age of onset in pubert y.
as if it were made of wax.

• Unlike the paranoid subtype of schizophrenia, delusions


and hallucinat ions are not t he most prominent
f e a t u r e , although fr agmen tary delusions and
hallucinations may be present.

• Individuals with catatonic schizophrenia of t en show


extreme immobilit y. They may stay in the same position
for hours, days, weeks, or longer. The position they assume
may be unusual and appear uncomfortable to the observer;
for example, the person may stand on one leg like a stork,
or hold one arm outstretched for a long time.

• The emotional responses of people diagnosed w ith


t h i s su b t y p e ca n of t e n se e m st r a n g e or
If an observer moves a hand or limb of the catatonic person’s
inappr opriat e to the situation. Inappropriate facial
body, he or she may maintain the new position. This
responses may be common and behavior is sometimes
condition is known as waxy flexibility.
described as ‘silly’.
• In other situations, a person with catatonic schizophrenia
may be extremely active, but t he activit y appears
bi zar r e, pur pose less, and unconn ect e d to the
situation or surroundings. Th e pa t ie nt m a y, f or
ex am pl e, r un up a nd dow n a f li ght of st ai r s
repeat edly.

• Complete lack of expressed emotion is sometimes seen,


as is an apparent indifference, anhedonia ( the lack
of pleasure) , and avolit ion ( a lack of mot ivation) .
D i sor g a n i z e d sch i z op h r e n i a char acte rized by • Some of these features are also present in other types
prominent disorganized behavior and speech , and of schizophrenia, but they are m ost prom inent in
flat or inappropriate emot ion and affect . Disorganized Schizophrenia.

• This form of schizophrenia is typically associated w ith


early onset ( oft en bet w een the ages of 15 and 25
years) and is thought to have a poor prognosis because
of the rapid development of ‘negat ive’ symptoms
and decline in social functioning.
• Disorganized schizophrenia is thought to be an extreme
expression of the ‘disorganization syndrome’ that
has been hypothesised to be one aspect of a three-factor
model of symptoms in schizophrenia

HELP LI NE NO. 9 39 1 56 7 70 7
TOPI C 1: SCHI ZOPHRENI A PSYCHI ATRY 3

• The other factors being ‘reality distort ion’ (involving • Features of retarded or st uporous catat onia -
delusions and hallucinat ions) and ‘psychomot or i) Mutism vi) Echolalia
poverty’ (poverty of speech, lack of spontaneous ii) Rigidity vii) Echopraxia
m ovement and var ious aspect s of blunt ing of iii) Negativism viii)Waxyflexibility
emot ion). iv) Posturing ix) Ambitendency
v) Stupor
5. Bad prognostic indicator of schizophrenia is : X) Mannerism stereotypes
A. Family history of schizophrenia XI) Automatic obedience
B. Late onset
C. Positive precipitating factors Features of excited catatonia -
D. Prominent affective - Increase in psychomotor activity e.g. restlessness agitation,
A excitement
.........( AI I MS PGMEE - FEB – 19 97) - Increase in speech production

9. Which symptom of schizophrenia respond earliest to


t reat ment
A. Negatavism
B. Paranoid delusions
C. Auditory hallucinations
D. Apathy
C
.........( AI I MS PGMEE - MAY - 199 3)

Posit ive sympt oms are f irst t o go aft er t he t / t of


6. Paranoid Schizophrenia is caused by - schizophrenia but none of them specifically mentions
A. L dopa auditory hallucination or even hallucination.
B. Fluoxetine Positive symptoms of schizophrenia include -
C. Amphetamine a) Hallucination
D. Ketamine b) Delusions
C c) Thought disorder
.........( AI I MS PGMEE - JUNE - 199 7)
• Drugs mainly used for the t/t of negative symptoms and
• Drugs, like LSD, amphetamines and mescaline, can cause resistant cases - Clozapine
schizophrenia-like symptoms in normal subjects.” Olanzapine
If extrapyramidal symptoms associated with drug therapy is
7. BAD prognostic factor for schizophrenia is to avoided -
A. Early onset of disease • Thioridazine
B. Catatonia • Clozapine
C. Presence of depression
D. Absence of family history 10 . Which is not classical 4A of Schizophrenia
A A. Ambivalence
.........( AI I MS PGMEE - JUNE - 199 7) B. Autism
C. Affect disorder
Prognostic Factors in Schizophrenia D. Automatism
D
Good Poor .........( AI I MS PGMEE - MAY - 199 4)
• Acute Onset Insidious onset
• Late Onset (Onset after 35 yrs • Early Onset (Onset before 20 Bleuer’s 4 A’s of schizophrenia -
age) yrs of age)
1) Ambivalence
• Presence ofppt stress • Chronic course 2) Autism
• First episode • Past history of schizophrenia 3) Affect disturbance
• Short duration ($.6 month) • Absence of stress 4) Association disturbance
• Female • Male
• Presence of depression • Absence of depression 11 . A pat ient complains that people living upstairs are
• Catatonic subtype • Flat or Blunt effect alw ays talking about him and conspire against him.
• Presence of confusion • Poor iocia/ support His son complain about his w eird behaviour like
• Good premorbid adjustment • Hospitalizaiton keeping shoes in fridge and w earing vest over his
• Family history of schizophrenia shirt. The likely diagnosis is:
A. Schizophrenia
8. The characterist ic feat ure of catatonic schizophrenia B. Depression
is C. Delusion disorder
A. Stupor D. Dementia
B. Rigidity A
C. Negativism .........( AI I MS PGMEE JUNE - 199 9)
D. All of the above
D I n Dementia there is no personality deterioration and the
.........( AI I MS PGMEE - MAY - 199 3) prominent symptom is memory loss.

HELP LI NE NO. 9 39 1 56 7 70 7
TOPI C 1: SCHI ZOPHRENI A PSYCHI ATRY 4

Depression can occur with Schizophrenia


• Negative symptoms are more resistant to treat so this
may be a possibility
• Reacting to external st imuli (i.e. being depressed in a
sad enviomment may be a possibility)
• Though Antipsychotic drugs can cause Parkinsonism the
pt. here does not give any symptom of Parkinsonism

1 3 . A 6 0 y r s. ol d m a l e su f f e r i n g f r om Au d i t or y
hallucination says t hat people staying upw ards are
talking about him and conspiring against him. He
dropped a police complaint against t hem but the
allegations w ere proved to be w rong. The diagnosis
is :
• I n delusion A. Depression
• 1. There is no gross impairment of personalit y . The B. Dementia
individual has near normal social occupat ional life, C. Delusional disorder
without arousing suspicion regarding disorder. D. Schizophrenia
• I t is only w hen areas of delusion is probed or D
conf r ont ed t hat per sonalit y disor ganisat ion is .........( AI I MS PGMEE MAY - 200 1)
evident .
• 2. Hallucination is not prominent in Delusion Symptoms of the pt. makes the diagnosis of schizophrenia
likely because
- Auditory hallucinations
III person hallucinations —characteristic of schizophrenia.
In Schizophrenia
Delusion of persecution also frequently occur.

Diagnosis can be confused with delusional disorder


it is ruled out as
- hallucinations are rare in delusion disorder
- never prominent.

• In Dementia
– memory loss is a prominent symptom.
• In Depression
– nihilistic delusion occurs
– most important feature is sadness of mood.
• Schizophrenia pt gives all of the Symptoms shown by
the pt such as 14 . Ramu a 22 yr old single unmarried man is suffering
1. IIIrd person Auditory Hullucinations. from sudden onset of 3rd person hallucination for
2. Gross personality impairment. t he past 2 w eeks. He is suspicious of his family
members and had decreased sleep and appetit e, t he
diagnosis is :
A. Acute mania
B. Acute psychosis
C. Delirium
D. Schizophrenia
B
.........( AI I MS PGMEE MAY - 200 1)

• For diagnosing schizophrenia 6 m on t h H/o Gross


disturbance in behaviour is needed and during this
period active symptoms of schizophrenia should be present
for at least 1 month..
• The Patient does not Satisfy any of the criteria so the
diagnosis of schizophrenia is ruled out.
12 . A pt. of schizophrenia on neuroieptics. His psychot ic
symptoms gets relieved but developed sadness, talks Acute psychosis vs Mania.
less t o others, remain on bed, all of t he follow ing • Acute Psychosis is a condition in which delusions
are likely causes, except: ,hallucinations, disorganized speech of grossly disorganized
A. Parkinsonism behaviour is present (the time limit is not specified)
B. Major depression
C. Negative symptoms are still persisting The symptoms of t he patient also indicate mania
D. He is reacting to external stimuli • Only 1 w k duration is required for Mania.
A • but during the period of mood dist urbance at least
.........( AI I MS PGMEE JUNE - 200 0) three symptoms ( out of the 7 required) should be
present.

HELP LI NE NO. 9 39 1 56 7 70 7
TOPI C 1: SCHI ZOPHRENI A PSYCHI ATRY 5

• The patient present w it h only one of the required – He is having delusions — wife being replaced by a double.
symptom (decreased sleep) This is k/a delusion of doubles.
• Presence of 3rd person hallucination also goes aginst mania • Both these points clearly m ak e t he di agnosis of
Paranoid schizophrenia .
• Symptoms required to diagnose mania
1. Inflated self esteem • The point “Pt , is w ell groomed and alert ” is of no help
2. Depressed need for sleep in differentiating between the two as both of them, the
3. More talkative than usual PPD and PSZ pat ients can be w ell groomed and
4. Flight of ideas alert.
5. Distractability
6. Increase in goal directed activity
7. Excessive involvement in pleasurable activities that have a
high potential for painful consequence (engaging in
unrest rained buying spree, sex ual indiscret ion
foolish business invest ment s)

15 . A 60 year old man is brought to a psychiat rist w ith


a 10 year history, t hat he suspect s his neighbours
and he feels that w henever, he passes by they sneeze
and plan against behind his back. He feels that his
w ife has been replaced by a double and calls police
for help, he is quit e w ell groomed alert , occasionaly
consumes alcohol. The likely diagnosis is :
A. Paranoid personality
B. Paranoid schizophrenia
C. Alcohol withdrawal 16 . A patient is brought w ith 6 month hist ory of odd
D. Conversion disorder behaviour. There is history of a family member having
B disappeared some years back. He seems to be talking
.........( AI I MS PGMEE MAY - 200 2) t o him self and somet im es m ut t er ing t o him self
loudly. The likely diagnosis is:
• I s it Paranoid personality disorder (PPD) vs Paranoid A. Schizophrenia
schizophrenia ( PSZ) B. Conversion disorder
C. Major depression
Both PPD and PSZ show the typical symptoms of D. Delusion
suspiciousness and mist rust of people. A
• There are tw o major differences between them .........( AI I MS PGMEE MAY - 200 2)
1) Absence of delusions and hallucinations in
PPD This is simply schizophrenia
2) PPD (or any personality disorder) have an
onset in adolescence or early adulthood . Conversion disorder
It’s a part of their personality • Is a disorder characterized by the presence of one or
• People w ill say that he w as like this from the very more neurological symptoms (for example paralysis,
beginning. blindness and paraesthesias) that cannot be explained by
a known neurological or medical disorder .
• Even though this pat ient is not m anif est ing t he
typical psychotic symptoms of schizophrenia (like
delusions, hallucinations; negative symptoms etc.) it is the
most likely diagnosis.
• There is a ver y w ide r ange of pr esent at ions of
Schizophrenic pat ient s from that of a com plet ely
dishelved, screaming, agit ated person t o an obsessively
groomed, complet ely silent and immobile person.

17 . The follow ing is a Schneider’s first rank symptom:


A. Persecutory delusion
• Whereas in Schizophrenia people will notice a drastic B. Voices commenting on actions
change in behaviour . Patient was alright and led a C. Delusion of guilt
completely normal life, after which he started behaving D. Incoherence
odd. B
• Personality disorders are excess of some traits which .........(AIIMS PGMEE MAY - 2003)
are normally distributed in the population. PGI - JUNE 2004, PGI - June -1998

• Coming to the question First rank sympt oms of schizophrenia: -


• This patient is 60 years old and t he history of his ( A) Hallucinations
odd behaviour is of 10 years , that means he w as (1) Audible thoughts
normal up t o 5 0 or 4 0 yrs of age after which his (2) Voices heard arguing
personality deteriorated. (3) Voices commenting on one’s action

HELP LI NE NO. 9 39 1 56 7 70 7
TOPI C 1: SCHI ZOPHRENI A PSYCHI ATRY 6

(B) Thought alienation phenomenon DISORDERS OF THE STREAM OF THOUGHT


(4) Thought withdrawal • In this category there is an alteration in either the amount
(5) Thought insertion or speed of thought.
(6) Thought diffusion or broadcasting • At one extreme there is pressure of thought, when ideas
(C) Passivity phenomenon arise in unusual variet y and abundance and pass
(7) Made feeling or affect through t he mind rapidly.
(8) Made impulses
(9) Made volition or acts
• At the other extreme there is poverty of thought, when
(10) Somatic passivity
the patient has only a few thoughts, which lack variety
(D) Delusional perception
(11) Delusional perception: Normal perception has a private and richness, and seem to move through the mind slowly.
and illogical meaning. The experience of pressure occurs in mania ; that of
The delusions included in first rank symptoms of schizophrenia povert y in depressive disorders.
are primary delusions also known as autoconthous • Either may be experienced in schizophrenia.
delusions. T hese delusions are char a cteristic of
schizophrenia. Pressure of speech

The secondary delusions are : • Stream of thought can be interrupted suddenly, the
(a) Delusion of persecution patient’s mind goes ‘blank’ and an observer may not ice
(b) Delusion of reference an int erruption in t he flow of conversation .
(c) Delusion of grandeur
• Minor degrees of this experience are common, particularly
(d) Delusion of control
in people w ho are t ired or anxious.
(e) Delusion of guilt.
These secondary delusions can be seen in schizophrenia but • In contrast, thought blocking, which is a particularly abrupt
these are not included in schiedner’s first rank and complete i nterruption, st r on g l y su g g e st s
symptoms of schizophrenia. schizophrenia

Eu g e n Bl e u l e r ’s Fu n d a m e n t a l Sy m p t om s of DISORDERS OF POSSESSION OF THOUGHT


Schizophrenia ( 4As) : 1) thought withdrawal
( 1 ) Ambivalence — severe inability to decide for or against 2) thought broadcasting
(2) Autism — withdrawal into self 3) Gedankenlautwerden
(3) Affect disturbance — Inappropriate affect • Patients with delusions of thought withdrawal believe that
(4) Association disturbances — loosening of associations, thoughts have been taken out of t heir mind
thought disorder.
– This delusion usually accompanies thought blocking.
18 . Lallo, 40 yrs. has recently st arted w riting books. But
the matter in this book could not be underst ood by • In delusions of thought broadcasting the patient believes
anybody since it contained w ords w hich w ere never that his unspoken thoughts are known to other people
there in dictionary and the t heme w as very disjoint. in some way.
Now a days he has become very shy and self absorbed
w h e n h e a d d r e sse s p e op l e h e sp e a k s a b ou t
metaphilosophical ideas. Likely diagnosis is:
A. Mania
B. Schizophrenia
C. A genius writer
D. Delusional disorder
B
.........( AI I MS PGMEE NOV - 200 0)

• The pt. is giving symptoms of schizophrenia. These are


• Words used by him were never there in any dictionary —
Neologism
• Metaphilosophical ideas Some people believe their thoughts can be heard by other
• The theme is v e r y d i sj oi n t a n d cou l d n ot b e people (Gedankenlautwerden).
understood by anybody All three of these symptoms occur more commonly in
Loosening of association & thought block. schizophrenia than in any other disorder.
• Very shy & self absorbed
– negative symptoms of schizophrenia.
• DISORDERS OF CONTENT OF THOUGHT
– Disorders of content of thought include delusions,
Disorders of thinking can be divided into the following
categories: preoccupations, obsessions, compulsions, phobias, plans,
– Disorders of the stream of thought ( speed and intentions, recurrent ideas about suicide or homicide,
pressure ) hypochondriacal symptoms and specific antisocial urges.
– Disorders of possession of thought – A major category of disorders of content of thought
– Disorders of content of thought includes delusions.
– Disorders of form of thought – They may be either mood congruent or mood incongruent

HELP LI NE NO. 9 39 1 56 7 70 7
TOPI C 1: SCHI ZOPHRENI A PSYCHI ATRY 7

Ideas of reference and ideas of influence should also be • Thus in response to a series of simple questions, the person
included in this section. may give the correct answer to the first question but
I deas of reference are e.g. the idea that one’s TV or continue to give the same answ er inappropriately
radio is speaking to or about one. to subsequent questions.
I deas of influence e.g. are beliefs involving another • Perseveration occurs in dementia but is not confined to
person or force controlling one’s behaviour. this condition

• Loosening of associations denotes a loss of the normal


Other disorders of content include overvalued ideas. structure of thinking.
• To the interviewer this appears as muddled and illogical
DISORDERS OF THE FORM OF THOUGHT conversat ion that cannot be understood by further
• This is conceptually the most difficult part of disorders of inquiry.
thought. – There are several types of this abnormality, but the striking
• One way of thinking about it is to consider that disorders feature in all of them is the general lack of clarity in the
of form of thought can be divided into three subgroups : patient’s conversation.
– Flight of ideas
– Perseveration • This muddled t hinking differs from those people with
– Loosening of associations low intelligence or anxiety.
– Anxious people give a more coherent account when they
• In flight of ideas the patients thought and conversation have been put at ease, while those with low intelligence
move quickly from one topic to another so that one train express their ideas more clearly if the quest ions are
of t hought is not completed before anot her one simplified.
appears. – When there is loosening of associations the more the
• These rapidly changing topics are understandable because interviewer tries to clarify matters the less he understands
the links betw een t hem are normal, a point w hich them.
different iates them from loosening of associations. – Loosening of associat i ons occurs most often in
schizophrenia.

Loosening of associations can take several forms:


• Knight’s move or derailment refers to a transition between
one topic and another, either mid-sentence or between
sentences, with no logical connection betw een t he t w o
and no evidence of such links that are found in flight
of ideas.

• In practice the distinction is often difficult to make,


especially when the patient is talking so rapidly.
• The characteristics of flight of ideas are:
– preservation of the normal logical sequence of ideas,
– using two words with a similar sound (clang association )
or the same word with a different meaning (punning ),
– rhyming and responding to the distracting cues in the
immediate surroundings.

Flight of ideas is characteristic of mania. • When this abnormality is extreme then it can disrupt
not only the connection between sentences and phrases,
but also the finer grammatical structure of speech.
I t is then called w ord salad.

• Perseveration is the persistent and inappropriate repetition


of the same thoughts.
• The disorder is detect ed by examining the person’s
w ords or act ions.

HELP LI NE NO. 9 39 1 56 7 70 7
TOPI C 1: SCHI ZOPHRENI A PSYCHI ATRY 8

• Verbigeration is when sounds, words or phrases are • Delirium tremens is an alcoholic withdrawal syndrome
repeated in a senseless way. It is a type of st ereotypy. seen in chronic alcoholics with C/F of confusion,
• One effect of loosening of associations is called vorbiereden disorientation & hallucination. The pt. does not give any
or talking past the point. The patient seems to get near history of Alcohol withdrawal & he is an occasional
to the point but never quit e reaches it. Alcoholic.
• Snapping off is the experience of a schizophrenic patient • In Delusion- hallucinations are not prominent. There are
when his chain of thought suddenly stops. It is not
no behaviour change.
caused by dist raction and the patient cannot give
any explanation for it. It is another name for thought
blocking. The patient may ex plain it in t er m s of 20 . Schizophrenia is characterized by all except
t hought w it hdraw al . A. Elation
B. Auditory hallucination
• Crowding of thought occurs in schizophrenia. His thoughts C. Catatonia
are passively concentrated and compressed in his head. D. Delusion
The patient may say that his thoughts are crowded into A
one part of his head it is a bit like flight of ideas but has .........( AI I MS PGMEE NOV - 200 0)
the schizophrenic quality of passivity.
• Elation is a feature of MANIA

21 . Loosening of association is an example of:


A. Formal thought disorder
B. Schneiderian first rank symptoms
C. Perseveration
D. Concrete thinking
A
• In derailment there is a breakdown in association so that .........( AI PGMEE - 200 6)
thoughts seem t o occur w ith no logical connection
bet w een them. The subject is unable to link his ideas • Loosening of associations
and there is a change in direction of his thinking. - Is a thought disorder
- Included in Bleuler’ s fundamental symptom of
Schizophrenia.
- Is a pattern of spontaneous speech in which things said
in juxtaposition lack a meaningful relationship. Speech is
described as being ‘disjointed.
• Perseveration
Persistent repetition of words beyond their relevance.
- Commonly seen in Schizophrenia
• In fusion there is some preservation of the normal chain
of associations, but there is a bringing t oget her of 2 2 . All of t he follow ing ar e associat ed w it h bet t er
heterogeneous element s. prognosis in schizophrenia, except
• Links are made in the speech which do not progress A. Late onset
clearly to the goal of thought B. Married
C. Negative symptoms
19 . a 24 yrs. old occasional Alcoholic had got a change D. Acute onset
in behaviour. He has become suspicious that people C
are trying t o conspire against him t hough his father .........( AI PGMEE - 200 6)
st at es t hat there is no reason for his fears. He is
getting hallucinations of voices commenting on his 23 . The t erm “Dement ia precox” w as coined by
action. W hat is most probable diagnosis : A. Freud
A. Delirium tremens B. Bleuler
B. Alcohol induced Psychosis C. Kraepelin
C. Schizophrenia D. Schineider
D. Delusional disorder C
C .........( AI PGMEE - 200 8)
.........( AI I MS PGMEE NOV - 200 0)
Emil Kraepelin coined term dementia precox.
• The symptoms of t he patient
• Change in behaviour
• Delusion of persecution
• IIIrd person halluination (Auditory)
• Marked change in personality
• They all point towards the diagnosis of shizophrenia.
• Alcholic psychosis or korsakoff’s psychosis presents
with Amnesia & Confabulation, pt. is not having both the
symptoms.

HELP LI NE NO. 9 39 1 56 7 70 7
TOPI C 1: SCHI ZOPHRENI A PSYCHI ATRY 9

24 . Perseveration is: Posturism : Voluntary assumption of bizzare, inappropriate


A.P ersistent and inappropriate repetition of the same posture for long period of time.
thoughts. Negativism: Motivates resistance to all commands and
B.When a patient feels very distressed about it. attempts to be moved.
C.Characteristic of schizophreniA) Echolalia : Repetition, echo or mimiking of phrases of
D.Characteristic of obsessive compulsive disorder (OCD) words heard
A 6. Echoproxia : repetition, echo or mimiking of actions
..........( AI PGMEE - 200 5) observed.
7. Waxy flexibility : Parts of body can be placed in positions
25 . A 16 year old boy does not at tend school because that will be maintained for long periods of time even if
of t he fear of being harmed by school mat es. He very uncomfortable – flexible like wax.
thinks that his classmates laugh at and t alk about 8. Mannerism
him. He is even scared of going out t o the market. 9. Aut om a t i c obe di a nce : commands are followed
He is most likely suffering from : automatically, irrespective of their nature
A. Anxiety neurosis 10. Ambit endancy : no good directed action
B. Manic Depressive Psychosis
C. Adjustment reaction 28 . Schizophrenia w ith lat e onset and good prognosis
D. Schizophrenia A. Simple SZP
D B. Hebephrenic
.........( AI PGMEE - 200 4) C. Catatonic SZP
D. Paranoid SZP
• Anxiet y neurosis & adjust ment reaction is easily C
ruled out as there is no stressor & delusions are present .........( AI PGMEE - 199 4)
& insight is lost.
• Manic depressive bipolar psychosis can be easily ruled out. Schizophrenia
• This boy is fulfilling 2 criterias of schizophrenia - disorganized Paranoid Hebephronic Catatonic Simple
behaviour & delusions, so the diagnosis is schizophrenia.
Occurs in later Occurs in Occurs in late Occur in early
life (3rd or 4th early 2nd 2nd or 3rd life (2nd
26. A patient of Schizophrenia is getting Chlorpromazine,
decade) decade decade decade)
but his audit ory allucinations are not controlled. The
next drug to be given is: Onset is Onset is Onset is Onset is
insiduous and insiduous usually acute insiduous and
A. Haloperidol
course is course is course is course is
B. Clozapine
progressive progressive episodic progressive
C. Sulpnde
D. Tianeptin Complete Worst Recovery from Prognosis is
B recovery prognosis episode is usually poor
usually does usually
.........( AI PGMEE - 200 0)
not occur complete
Clozapine is an atypical antipsychotic drug. 29 . Basic difference betw een neurosis and psychosis is
It suppresses both positive and negative symptoms of A. Severity
schizophronia ad many patients, who were refactory to B. Insight
typical neuroleptics, respond t o Clozapine. C. Clinical features
It should however be used any as reserve drug in resistant D. Duration
schizophrenia B
Major limitation is : higher incidence of agranulocytosis. .........( AI PGMEE - 199 5)
Major advantage is: no extra- pyramidal side effects.
Features Psychosis Neurosis
27 . A pat ient present w ith w axy flexibility, negativitism
Judgement Impared Preserved
and rigidity. Diagnosis is
A. Catatonic schizophrenia Insight Lost Preserved
B. Paranoid schizophrenia Personality Disturbed Relatively preserved
C. Hebephrenic schizophrenia Contact with reality Impared Preserved
D. Simple schizophrenia Characteristic Delusions and Symptoms cause subjective
A symptoms hallucinations distress because they are
.........( AI PGMEE - 200 1) recognized as undesirable
(insight present)
Catatonic schizophrenia is char acterised by m ar k e d Examples Schizophrenia Anxiety OCD Conversion
dist ur bance of mot or behavior in addition to the Mood disorders disorder Phobia
general features of schizophrenia.
The patient in quest ion present w ith typical signs 30 . I n Schizophrenia early onset w it h poor prognosis is
of r et ar ded cat at onia, and hence diagnosis is seen in
catatonic shizophrenia. A. Simple
B. Hebcphrenic
Characteristics catatonic signs C. Catatonic
Mutism: Absence of speech D. Paranoid
Rigidit y : Maintenance of Rigid posture against efforts B
to be moved .........( AI PGMEE - 199 6)

HELP LI NE NO. 9 39 1 56 7 70 7
TOPI C 1: SCHI ZOPHRENI A PSYCHI ATRY 10

• Hebephrenic schizophrenia has early onset and one of lacks insight to his changed behaviour and symptoms.
the worst prognosis (only better than simple type.). The Likely diagnosis is:
recovery from episode classicaly never occurs. A. Psychosis
B. Schizophrenia
Schizophrenia C. Paronia
Common Ampheta Early onset Late Very late Simple D. Depression
est type mine & bad onset (3rd, 4th A
Causes prognosis Best decade) .........( AI PGMEE - 199 7)
Prognosis
Paranoid Paranoid Hebephrenic Catatonic Paranoid Simple
3 1 . A pat ient of schizophr enia t r eat ed f or 5 year s
develope perioral movement s. Likely diagnosis is:
A. Tardive dyskinesia
B. Muscular dystonia
C. Akathisia
D. Malignant neuroleptic syndrome
Psychosis is defined by the following characterstics :
A
1 Gross impairment in reality testing (contact with reality)
.........( AI PGMEE - 199 7)
2 Marked disturbance in personality with impairment in social,
interpersonal and occupational functioning.
• Tardive dyskinesia is the most common delayed ( lat e) 3 Marked impa irment in judgement and absent
complicat ion of antipsychot ic medication understanding of current symptoms and behaviour (loss
• It is characterised by purposeless, involuntary facial and of insight)
limb movements 4 Presence of characteristic symptoms like
1-delusions
- hallucinations

Tardive Dyskenysis
• Muscular dystonia :
– is characterised by bizzare muscle spasms specially affecting
Linguofascial muscles
– Grimacing etc.occurs within few hours of a single dose at
most most within a week. The patient in question presents with the characteristics
symptoms of
- delusions &
- hallucinations
Also, insight t o his current behaviour and symptoms is
not present.
The diagnosis is thus consistent with the definition of psychosis

Muscular dystonia
Thus w hile facial movements can be a feature of both tardive
dyskinesia and muscular dystonia, those developing as
late as 5 years after institut ion of treat ment are
likely to be due to be due to tardive dyskinesia.
• Akathasia : Refers to r e st le ssne ss / f ee li ng of
discomfort / apparent agitation facial movements are
not a feature.
33 . ‘First order ‘symptoms of schneiders’ schizophrenia
Akathasia
include all except:
• Malignant neuroleptic syndrome : is charactrised by Rigidity,
A. Deparsonilization
immobility, fever, fluctuating BP and heart rate.
B. Running commentery of ones thoughts
Facial movements are not a feature.
C. Primary delusion
D. Somatic passivity
3 2 . A pat ient present s w it h a one mont h hist or y of
A
abnormal hallucinat ion, and delusion. The patient
.........( AI PGMEE - 199 8)

HELP LI NE NO. 9 39 1 56 7 70 7
TOPI C 1: SCHI ZOPHRENI A PSYCHI ATRY 11

Schneider’s first rank symptoms of schizophrenia include : 38 . Drug treatment of schizophrenia are :
A. Hallucinations: A. Chlodiazepoxide.
1. Audible thoughts B. Trifluperazine.
2. Voices heard arguing C. Clozapine.
3. Voices commenting on one’s action D. Rivasligmine.
B. Thought Aleniation Phenomenon: E. Haloperidol
1. Thought withdrawl B, C & E
2. Thought insertion ..........( PGI - DEC 200 3)
3. Thought diffusion or broad casting
C.Passivity: • Rivastigmine is used in Alzheimer’s disease.
1. ‘Made’ feelings or affect • Chlordiazepoxide is a benzodiazepine.
2. ‘Made’ impulses
3.’Made’volition 39 . Most common t ype of schizophrenia
‘Made’ here refers to ‘being imposed by some ext ernal A. Hebephrenic
force’. B. Catatonic
4. Somatic passivity : Body sensations are experienced as C. Paranoid
imposed on body by some external force. D. Un differentiated
D. Delusional perception: normal perception has a private C
and illogical meaning .........( PGI - DEC 200 4)
• First rank symptoms are considered to be pathognomonic 40. Biochemical abnormalities in schizoohremia:
for schizop hrenia in the absence of organic A. Increased Dopamine
psychopathology. They are helpful but w ill yield many B. Increased dopamine and Increased serotonin
false positives. C. Decreased Dopamine
• They should not be used as absolute criteria for diagnosis. D. Increased Dopamine and decreased serotonin
A& B
34. Neurotransmit ter related to schizophrenia pathology .........(PGI - DEC 2002)
is 41 . I n schizophrenia, charact eristic feature is:
A. Ach A. Formal thought disorder
B. Dopamine
B. Delusion
C. Serotonin
C. Hallucination
D. NA
D. Apathy
A, B & C
A
.........( PGI - 1 997 - Dec)
.........( PGI - JUNE 199 7)
Schizophrenia is presently thought to be probably due to a
• Schizophrenia is characterized by disturbances in thought
functional increase of dopamine at the post-synaptic
receptor, though other neurotransmistters like serotonin & verbal behaviour, perception, affect, motor behaviour
(especially 5HT2 receptors), GABA & choline acetyl (Ach) & relationship to the external world.
are also presumably involved. Of different types of delusions and hallucinations prevalent
in schirophrenia, primary, delusions are characteristic of
35 . I mpaired insight is found in: schizophrenia and only third person hallucinations are
A. Traumatic psychosis characteristic of schizophrenia.
B. Schizophrenia
C. Anxiety neurosis 42 . Delusions of control, persecution and self reference
D. Obsessive compulsive neurosis are seen in:
B A. Paranoia
.........( PGI - 1 997 - Dec) B. Paranoid schizophrenia
C. Mania
• In schizophrenia, insight is absent & judgement is usually D. OCD
poor. B
In mania also insight into the illness is absent. .........( PGI - JUNE 199 7)
Paranoid schizophrenia is characterized by the following
36 . Good prognosis in schizophrenia is indicat ed by: features, in addition to the gener al guidelines of
A. Soft neurological signs schizophrenia:
B. Affective symptoms • Delusions of persecution, reference, grandeur (or
C. Emotional blunting grandiosity), control or infidelity (or jealousy). The delusions
D. Insidious onset are usually w ell-systematized ( ue. w ell connect ed
B w ith each ot her) .
.........( PGI - 1 998 - Dec) • The hallucination usually have a persecutory or grandiose
content.
37 . Schneider’s first rank symptoms I n schizophrenia • Disturbancs of affect, volition, speech & motor behavior.
include The personalit y deterioration is much less than other
A. Audible thoughts type of schizophrenias.
B. Somatic passivity
C. Derealization 43 . True about schizophrenia :
D. Depersonalization A. Thought broadcasting
E. Hallucinations B. Third person hallucination
A, B & E C. Makes violence
.........( PGI - 2 001 - Dec) D. Elated mood

HELP LI NE NO. 9 39 1 56 7 70 7
TOPI C 1: SCHI ZOPHRENI A PSYCHI ATRY 12

E. Takes good self care 48 . Drugs used in schizophrenia include :


A, B & C A. Chlorpromazine
.........( PGI - JUNE 200 3) B. Halopendol
45 . Follow ing are correctly matched except: C. Olanzapine
A. Schizophrenia - Audiory hallucination D. Imipramine
B. OCD - infidelity E. Risperidone
C. Alcoholism - auditory hallucination A, B, C & E
D. Depression - guilt feeling ..........( PGI - June -200 2)
B • Imipramine is an anti-depressant.
.........( PGI - June -199 8) • Rest of the drugs mentioned are used in Schizophrenia.

• In obsessive compulsive reaction, the irrational idea or 49 . Schizophrenia and depression both have A/ E
impulse persistently intrudes into awareness. A. Formal thought disorder
Obsessions (constantly recurring thoughts such as fears of B. Social withdrawal
exposure to germ) and compulsions (repetitive actions C. Poor personal care
such as washing hands before feeling a potato) are D. Decreased interest in sex
recognized by individual as absurd and are resisted. Failure E. Suicidal tendency
of which may lead to distress. A
• 2/3rds of OCD patient will develop major depression in .........( PGI - June -200 2)
their life-time. • Formal thought disorder : Disturbance in the form of
Under extreme stress patients sometimes exhibit paranoid thought, instead of content of thought; thinking
and delusional behaviour, often associated with depression, characterised by loosened associations, neologisms, and
and can mimic schizophrenia , infidelity is unlikely. illogical constructs, though process is disordered and the
• Hallucinations in schizophrenia are common- they are person is defined as Psychotic.
thought echo, third and 2nd person hallucination. • Schizophrenia is a thought disorder, but depression is a
• In depression— there is low mood, low self esteem, guilt- Mood disorder.
feeling, worthlessness, helplessness, hopelessness, suicidal • Affect ive symptoms like apathy, suicidal tendencies,
ideation etc. anhedonia is common in both the conditions.
• Negat ive sym pt om s like-apathy, social withdrawal,
47. Which t s more appropriat e in a case of schizophrenia affective flattening or blunting, alogia and attentional
A. Low socioeconomlc group impairment can occur in both conditions.
B. Seen in adolescents • Decreased motor activity, slowness, decreased self care,
decreased interest in sex can occur in both the conditions.
C. Common in primitive societies
D. Affluent society influences the incidence
50 . A 23- year-old engineering student is brought by his
B
family to the hospital w it h history of gradual onset
.........( PGI - June –200 0)
of suspiciousness, mutt ering and smiling w it hout
• Though all the factors mentioned here are risks associated
cle a r r ea son , d e cr e ase d soci al i za t ion , v iol en t
with schizophrenia the onset in adolescent is more
out bur st s, and lack of int er est in st udies f or 8
appropriate.
months. Mental status examinat ion revealed a blunt
• Schizophrenia commonly begins in late adolescence, has effect , t hought broadcast , a relat ively preserved
on insidious onset and classicaly a poor outcome. cognition, impaired judgement and insight. He is most
RISK FACTORS for schizophrenia. likely to be suffering from
• There are three principal risk factors A. Delusional disorder
— Genetic susceptibility B. Depression
— Early developmental insults like C. Schizophrenia
Rh factor incompatibility, Prenatal exposure to influ enza D. Anxiety disorder
virus in 2nd trimester, Prenatal nutritional deficiency. C
— Winter birth. .........( AI I MS PGMEE - MAY 200 4)
Sch i z op h r e n i a is commo n with family hi story of DSMI V crit ieria for diagnosis of Schizophrenia -( a)
schizoaffective disorder and schizotypal and schizoid Characteristics symptoms -2 or more of the following,
personality disorder. each present for a significant period during a 1 month
• Neurotransmitters involved in schizophenia : period.
— Dopamine in mesolimbic and mesocortical areas 1. Delusions
— Serotonin 2. Hallucinations
— Acetylcholine, glutamate, GABA 3. Disorganised speech (e.g. frequent der ailment or
• Cerebral damage in schizophrenia : incoherence)
— Enlargement of lateral and third ventricle 4. Crossly disorganized or catatonic behaviour
— Cortical atrophy 5. Negative symptoms (e.g. affective flattening, alogia or
— Sulcal enlargement avolitiori)
— Volumetric reductions in amygdala, hippocampus, right
prefrontal cortex, thalamus (b) Social/ Occupational dysfunction -
— Decreased metabolism in thalamus, prefrontal cortex For a significant portion of the time since the onset of the
— Altered asymmetry of planum temporale. disturbance one or the major areas of function such as
• Schizophrenia is more common in lower socio-economic work, interpersonal relations, selfcare are markedly below
group, family stress (specially in expressed emotions) the level achieved prior to the onset (or when the onset
• Ego defense mechanisms in schizophrenia — Denial is in childhood or adolescence, failure to achieve, expected
— Projection level of int erpersonal, occupation or academic
— Reaction formation achievement.)

HELP LI NE NO. 9 39 1 56 7 70 7
TOPI C 2: DEPRESSI ON PSYCHI ATRY 13

51 . Schizophrenia false is Negative symptoms are so-named because they are considered
A. 3rd person auditory hallucination to be the loss or absence of normal t rait s or abilities,
B. Inappropriate emotions and include features such as flat or blunted affect and
C. Sustained mood changes emotion, poverty of speech (alogia), anhedonia, and lack
D. Formal thought disorder of motivation (avolition).
C
.........( AI I MS PGMEE - NOV 200 7)

• Third person hallucinations are auditory hallucinations


in which patients hear voices talking about themselves,
referring to them in the third person, for example “he
is an evil person”.
• This type of auditory hallucination is par t icula r ly
associat ed w it h schizophr enia , but can occur in
affective disorders.
• Such voices may be experienced as comment ing on
the patient ’s intended actions - “he w ants to kill A third symptom grouping, the d i sor g a n i z at i on
her”, or describing his current actions - “he is trying syndrome, is commonly described, and includes chaotic
to sleep now ”. A running commentary by voices is most speech, thought, and behaviour.
suggestive of schizophrenia.

• Se con d or d e r h a l l u ci n a t i on s are a u d i t or y
hallucinations in w hich a voice appears t o address
the patient in the second person .
• For example the voice may be talking directly to the patient
- “You are going to die ” - or the voice may be telling
the patient to do some action - “kill him”. These types
of auditory hallucinations are not diagnostic in the same
way as third person auditory hallucinations, but the content 53. Neologism are characteristically seen in
of the hallucination, and the patient’s reaction to it, may A. Depression
help in diagnosis. B. Mania
• In a depressive psychosis the comments from t he C. Schizophrenia
audit ory hallucination may be derisory ( “you are D. Dysphasia
useless”) , and the patient may accept them as being C
justified. .........( AI I MS PGMEE - DEC 199 8)
• A schizophrenic may ex perience second person • N eologism Newly formed words or phrases whose
hallucinations but may resent the comments that t he derivations cannot be understood.
voice makes. • This is characteristically seen in Schizophrenia.
– These i n t e r p r e t a t i on s of t h e con t e n t of t h e • Blamestorming,Brainstorming
hallucinat ion and the pat ient ’s react ion are only • Treew are Hardw are,Softw are,Coursew are
indicators to the possible psychiatric diagnosis. • Web surfing
• I rritainment ,Edutainment
52 . Schizophrenia is characterized by all Except • horizontical
A. Delusion • confust ion
B. Auditory hallucination – Confusion, frustration fusion.
C. Elation in-a-gadda-da-meeting
D. Catatonia – Any meeting that could have been completed in half the
C time it actually took.
.........( AI I MS PGMEE - JUNE 199 8)
TOPI C 2 : DEPRESSI ON
Elation is a feature of mania
Positive and negative symptoms 54 . A 41 -year-old w oman presented w ith a hist ory of
Schizophrenia is often described in terms of posit ive (or aches and pains all over t he body and generalized
productive) and negative (or deficit) symptoms w eakness for four years. She cannot sleep because
P ositive sympt oms include d e l u si on s, a u d i t or y of the illness and has lost her appet ite as w ell. She
hallucinations, and thought disorder , and are typically has lack of inter est in w ork and doesn’t like t o meet
regarded as manifestations of psychosis. friends and relatives. She denies feelings of sadness.
Her most likely diagnosis is:
A. Somatoform pain disorder.
B. Major depression.
C. Somatization disorder.
D. Dissociative disorder
B
..........( AI I MS PGMEE NOV - 200 2)
Major depression
Almost half of t he pts of depression deny sadness
of mood

HELP LI NE NO. 9 39 1 56 7 70 7
TOPI C 2: DEPRESSI ON PSYCHI ATRY 14

Presence of either one of the following two symptoms along


with other symptoms is required for the diagnosis of
depression
(a) Depressed mood most of the day (feels sad or empty)
(b) Markedly diminished interest or pleasure in almost all
activities most of the day

Diagnosis of Somatized depression


•Significant decrease in weight or appetite
•Early morning awakening at least 2 hours before usual time
•Diurnal variation with depression being worst in the morning
•Pervasive loss of interest & loss of reactivity to pleasurable
stimuli

Other symptoms which favours depression in this patient


(a) Insomnia due to illness
(b) Decreased appetite.
• Patients w it h milder form of depression might deny
experiencing sad moods and instead complaints of
physical pain from headache, epigastric pain and so on,
in the absence of any diagnosable physical illness .
Such conditions are described as ‘depression sine
depressione’ or masked depression such cases are Depressive syndrome which do not present with classical
commonly observed in older patients”. features of depression:
• Depression w ith predominant anxiety
• Phobic-anxiet y-depersonalization syndrome ( PAD
syndrome)
- Common in women aged 20-40 years
- Characterized by diffuse anxiety, multiple phobias, panic
attack, depersonalization & depressive features
• Non endogenous depression
- Absence of neurotic traits
- Sigiificant stressful iife events
• Hysteroid- dysphoric depression
- Depression with marked anxiety, Pallor and changes in
55 . I ntense nihilisim, somatization and agitation in old physical appearance
age are t he hallmark symptoms of: - Person usually has histrionic (hysterical) personality traits
A. Involutional melancholia
B. Atypical depression Involutional Melancholia
C. Somatized depression • Form of severe depression
D. Depressive stupor • Occurs in involutional period (40- 65 years of age)
A • There is usually no previous history of Mental illness
.........( AI PGMEE - 200 6) • Characterized by -
- Depression
I n old age ,presence of depression, delusions of sin, guilt, - Delusion of persecution, sin, guilt, poverty
poverty, persecution, intense nihilism , somatization with - An obsession with death (Nihilism)
marked agitation A dejection ~ favours diagnosis of - Multiple Somatic Symptoms (imagined gastrointestinal
involutional melanocholia disease)
- Marked agitation & dejection

Diagnosis of stupourous depression

- In Younger patient < 40 year old.

- An extremely depressed phase of Manic-depressive psychosis 56 . An 18 year old student complains of lack of interest
characterized by extreme psychomotor retardation and in studies for last 6 months. He has frequent quarrels
unresponsiveness to surrounding condition . w it h his parents and has frequent headaches.The
most appropriate clinical approach w ould be t o:

HELP LI NE NO. 9 39 1 56 7 70 7
TOPI C 2: DEPRESSI ON PSYCHI ATRY 15

A. Leave him as normal adolescent problem. Lack of int er est in school is a k now n feat ur e
B. Rule out depression. ( anhedonia) .
C. Rule out migraine. Restlessness & irritability are part of dysphoric mood &
D. Rule out an oppositional defiant disorder explain frequent quarrels with parents.
B Also somatic symptoms like headache/ heaviness of
..........( AI PGMEE - 200 5) head are know n associations.

The patients behavior can neither be regarded as normal Clinical


manifestations of
adolescent behavior nor can headache be sufficient reason depression in
to classify the disorder as migraine. The main differential children and
diagnosis rests bet w een depression & oppositional adolescents
defiant disorder Anhedonia Loss of interest and enthusiasm in play ,
socializing, school, and usual activities;
boredom; loss of pleasure
Diagnostic criteria for oppositional defiant disorder
Dysphoric mood T earfulness; sad, down turned expression;
A. A pattern of negativistic, hostile, and defiant behaviour slumped posture; quick temper; irritability;
lasting for at least 6 months, during which four (or more) anger
of the following are present- Fatigability Lethargy and tiredness; no play after school
1. Often loses temper Morbid ideation Self-deprecating thoughts, statements;
2. Oft en argues w ith adults thoughts of disaster, abandonment, death,
suicide, or hopelessness.
3. Often actively defies or refuses to comply with
Somatic symptoms Changes in sleep and appetite patterns;
adults’ requests or rules difficulty in concentrating; bodily complaints,
particularly

57 . Neurotransmittors involved in depression are


A. GABA and Dopamine
B. Serotonin and Norepineprine
C. Serotonin and Dopamine
D. Norepinephrine and GABA
B
.........( AI PGMEE - 199 5)

4. Often deliberately annoys people Mood disorders are presently thought to be probably due to
5. Often blames others for his or her mistakes or misbehaviour functional increase or decrease of amines at the synaptic
6. Is often touchy or easily annoyed by others cleft (Bio-chemical theory).
7. Is often spiteful and resentful
8. Is often spiteful or vindictive Depression Decreased Nerepinephrine
Not e: Consider a crit erion met only if t he behaviour and serotonin levels.
occurs more frequently than is typically observed in
individuals of comparable age and developmental level. Mania Increased Norepinephrine
and serotonin levels.
5 8 . All of t he follow ing are r isk fact ors for suicidal
tendency in patients w it h depression except:
A.Females : age<40; unmarried, divorced or widowed
B. Written or verbal communication of suicidal intent
C.Early stage of depression
D.Recovering stage of depression
A
.........( AI PGMEE - 199 6)

B. The disturbance in behaviour causes clinically The risk of suicide in pat ient s w it h depression is
significant impairment in social, academic, or increased in presence of follow ing factors :
occupational functioning. • Presence of marked hopelessness.
C. The behaviours do not occur exclusively during the course • Males; age > 40; unmarried, divorced or widowed.
of a psychotic or mood disorder. • Written of verbal communication of suicidal intent.
• Criteria are not met for conduct disorder, and if the • Early stages of depression.
individual is age 18 years or older, criteria are not met for • Recovering stages of depression.
antisocial personality disorder. • Period of 3 months from recovery.

The presence of frequent headaches and age of


present at ion ( 1 8 years) make t he diagnosis of
oppositional delusional disorders unlikely.

Ruling out depression in this patient is therefore t he


most viable option
Depression can occur at any age including adolescents
and young adults.

HELP LI NE NO. 9 39 1 56 7 70 7
TOPI C 2: DEPRESSI ON PSYCHI ATRY 16

59. Delusions of nihilism and early morning insominia are concentration & reduce intellectual capacity.
characteristic features of: In contrast, a patient of dementia does not have these
A. Major depresion disturbance. In fact when confronted with the evidence
B. Schizophrenia of memory impairment, he often confabulates.
C. Mania
D. Personality disorder • Hyst erical pseudodement ia ( Ganser’s syndrome) is
A commoniy found in prison inmates. The characteristic
.........( AI PGMEE - 199 7) feature is v orbeireden which is al so called as
“approximat e answ ers”.
Depression is associated with sadness of mood. Sadness of
mood inturn is associated with pessimism.
This results in ideation such as that of nihilism i.e. ‘n ihilist ic
delusions’ such as :
1. Worthlessness
2. Helplessness
3. Hopelessness

Early morning insomnia characterised by ‘early morning


awakening’ at least 2 hrs. before the usual time of waking
up is characteristic feature of the somatic syndrome in
depression. • Ganser syndrome is a rare dissociative disorder previously
classified as a factitious disorder. It is characterized by
nonsensical or w rong answ ers to questions or doing
things incorrectly, other dissociative symptoms such
as fugue, amnesia or conversion disorder, often
w ith visual pseudohallucinations and a decreased
st ate of consciousness.
• It is also sometimes called non sense sy ndr om e,
60 . Most common age for depression is: balderdash syndrome, syndrome of approximat e
A. Middle age men answ ers, pseudodementia or prison psychosis .
B. Middle age female • This last name, prison psychosis, is sometimes used
C. Young girl because the syndrome occurs most frequently in prison
D. Children inmat es, where it may represent an attempt to gain
B leniency from prison or court officials.
.........( AI PGMEE - 199 8)
• Ganser is an extremely rare variation of dissociative
Depression is more common in two age groups : disorder. It is a react ion to ext reme st ress and t he
- late third decade pat ient t hereby suf fers fr om approx im at ion or
- fifth to sixth decade giving absurd answ ers t o simple questions . The
It is common in females (20-26 years compared to 8-12%) syndrome can sometimes be diagnosed as merely
Thus most common age of presentation is “ middle to old malingering, however, it is more often defined as
aged females” dissociative disorder.

• Symptoms include a clouding of consciousness, somatic


conversion symptoms, confusion, stress, loss of personal
identity, echolalia, and echopraxia. The psychological
symptoms generally resemble the patient’s sense of mental
illness rather than any recognized category. Individuals also
give approximate answers to simple questions. For
example, “How many legs are on a cat ?”, to w hich
the subject may respond ‘3 ’.

• The syndrome may occur in persons with other mental


61 . Pseudo dementia is seen in: disorders such as schizophrenia, depressive disorders, toxic
A. Alcoholism states, paresis, alcohol use disorders and factitious
B. Depression disorders. EEG data does not suggest any specific organic
C. Schizophrenia cause
D. Mania
B 62. Suicide risk is common w ith w hich type of depression
.........( PGI - 1 997 - Dec) :
A. Reactive depression
• Pseudodementia is found in depression. Depression in B. Endogenous depression
elderly patient may present as dementia clinically. It is called C. Childhood depression
depressive pseudodementia. The identification of D. Depression in involution
depression is very important as it is very often treatable. B& D
These patients themselves complains of memory .........( PGI - 1 999 - Dec)
impairment, difficulty in sustaining attention &

HELP LI NE NO. 9 39 1 56 7 70 7
TOPI C 2: DEPRESSI ON PSYCHI ATRY 17

• Reactive depression is a self-limiting condition. 90% after three episodes.


• Childhood depression - can cause suicide but more About 20- 35% cases are refractory to t he t reatment.
prevalent w hen associated w it h conduct
disorder or substance abuse.
I nvolut ional depression - occur in older age group,
vegetative and hypochondriacal feature can occur.

6 5 . The follow ing are t he psychiat ric sequalae aft er


st roke in elderly :
A. Depression
B. Post traumatic stress disorder.
63 . “Nihilist ic delusions” are seen in : C. Dementia.
A. Endogenous depression D. Apathy
B. Double depression
C. Depression in involutional stage A& D
D. Cyclothymia .........(PGI - DEC 2003)
E. Dysthymia
A& B Following are the sequence of stroke in elderly.
.........( PGI - 2 000 - Dec) - Vascular dementia
- Major depressive disorder or depression
• Nihilistic delusions e.g, “World is coming t o end ; there - Mania
is no brain in skull; intestines have - Anxiety
rooted away” with early morning insommia is typical of - Psychosis
endogenous major depression. - Apathy
• Double depression is a major depressive episode - Catastrophic reaction.
superimposed on an underlying dysthymia
or neurotic depression (usually chronic) 66 . True about psychotic feature in depression
• Nihilism also found in schizophrenia. A. Found in Severe depression
B. Found in Moderate depression
C. Mood incongruent psychotic feature
D. Clyclothymia
E. Dysthymia
C
.........( PGI - DEC 200 4)

> Presence of psychotic features in major depressive


disorder reflects severe disease and is poor prognostic
indicator.
64 . True about major depressive dLorder : Psychotic features are:
A. Commonly seen in female. • Mood congruent (mood not in harmony with mood
B. Recovery is complete after treatment. disorder)
C. Associated with hypothyroidism. • Mood incongruent (mood in harmony with mood disorder)
D. Family H/O major depression
A&D • m ood- in congr uent psy chosis has del usions or
..........( PGI - DEC 200 3) hallucinat ions w hose cont ent does not involve
typical depressive themes of personal inadequacy, guilt,
• The diagnosis of major depressive disorder (MDD) reuries disease, death, nihilism, or deserved punishment.
the presence of one or more major depressive episodes • Included here are such symptoms as per secut or y
and absence of any manic, hypomanic or mixed epsodes. de lusions ( not d ir ect ly r el at ed t o dep r e ssi ve
Their symptoms should not due to direct effect of themes) , thought insert ion, thought broadcasting,
substance or a gener al medical con dition e.g. and delusions of being possessed or under t he
hypothyroidism. control of an alien being
• Approx 50% of individuals with MDD have a first degree
relative with a mood disorder, which is more oft en 67 . I ncidence of suicide :
depression rather than bipolar disorder. A. 8-I0\ 1000 population
• The overall prevalence of MDD in woman is roughly twice B. 8-10\10,000 population
the prevalence in man; the mean age of onset for MDD is C. 8-10 per lac population
mid 20s. D. 8-10 per 10 lac population
Recurrence rate of MDD : E. 8-I0 per billion population
50% after one episode C
70% after two episodes .........(PGI - JUNE 2005)

HELP LI NE NO. 9 39 1 56 7 70 7
TOPI C 2: DEPRESSI ON PSYCHI ATRY 18

- The official suici de r ate in India is 9 . 9 / l a k h • I ncreased appetit e and w eight gain is seen in TCAs,
population\ year. but not with SSRI. (e.g. fluoxetine) and bupropion.
- The compar able period prevalence rate for suicide • Thioridazine has highest propensity for anti-cholinergic
throughout the world r anges fro m 5-30\lakh side effects.
population\year. • BZDs have low er abuse liability than barbiturates ;
tolerance mild, psychological and physical dependence and
68 . Suicidal tendencies are most common I n : withdrawal symptoms less marked .
A. Involutional depression
B. Reactive depression
C. Psychotic depression
D. Childhood depression
C
.........( PGI - June -200 0)
• Suicide is most common in major depression and also
common in psychotic depression.

69 . Suicidal tendency seen in : .


A. Schizophrenia
B. PTSD 71. A 25-year-old w oman complains of intense depressed
C. Depression mood for 6 months w ith inability to enjoy previously
D. OCD pleasurable activit ies. This symptom is know n as
E. Anxiety disorder A. Anhedonia
A, B & C B. Avolition
.........( PGI - June -200 2) C. Apathy
D. Amotivation
• Common causes of suicide : A
• Psychiatric disorders : These are the major causes of .........( AI I MS PGMEE - MAY 200 5)
suicide.
Some common causes are: • Anhedonia inability to experience pleasure in previously
- Depression : pleasurable activities. It is often asociated with depression.
— Major depression • Avolition lack of initiative
— Depression secondary to serious physical illness • Amotivation reduced drive and lack of ambition.
— Reactive depression, secondary to life stressors • Apathy is the absence or lack of feeling, emotion, interest,
- Alcohol and drug dependance concern, or motivation
- Schizophrenia
(ii) Physical disorders : Patients with incurable or painful
physical disorder like cancers and AIDS, often commit
suicide.
(iii) Psychosocial factors : Like failure in examination, failure
in love, dowry difficulties, marital difficulties, loss of loved
object, death etc.
Post-traumatic stress disorders (PTSD) occasionally leads to
suicidal ideation.
72 . A 50- year-old male present s w ith a 3 -year hist ory of
70 . Select the t rue statements : irritability, low mood, lack of interest in surroundings
A. Imipramine is used in treatment of endogenous depression and general dissatisfaction w ith everything. There is
B. Fluoxet ine causes w eight gain no significant disruption in his sleep or appetite. He
C. Thioridazine cause less anticholinergic effects is likely to be suffering from:
D. BZD’s have same abuse potential as barbiturates A. Major depression
E. Codeine is an effective antitussive B. No psychiatric disorder
A& E C. Dysthymia
.........( PGI - June -200 2) D. Chronic fatigue syndrome
C
• I m ipr amine, an ant i- depr essant is indicat ed in .........( AI I MS PGMEE - MAY 200 5)
depressive episode or endogeneous depression.
• Fluoxetine causes weight loss • It is case of dysthymic disorder.

• This disorder is very similar to major depressive disorder.


So, before establishing the diagnosis of
Dysthymia, diagnosis of maj or depressive disorder
must first be ruled out.

• To meet the diagnostic criteria of dysthymia, a patient


should not have symptoms that are better accounted
• Anti-cholinergic side-effect s like mydriasis, impaired for as major depressive disorder and the patients should
ejaculation is maximum with thioridazine not have maniac or hypomaniac episodes.

HELP LI NE NO. 9 39 1 56 7 70 7
TOPI C 2: DEPRESSI ON PSYCHI ATRY 19

D. GABA
B
.........( AI I MS PGMEE - MAY 200 8)

Suicide is caused by diminished serotonin level


Serotonin is a hormone that acts as neurotransmitter.
Serotonin levels in our body affects a wide range of mental
and physical responses and plays a significant role in
activities such as temper ature regulation, longer
sensations, sexual behaviour and sleep patterns.
Serotonin receptors also help to regulate emotions,
The profile of dysthymic disorder overlaps with that of major cognition, pain and sensory motor function.
depressive disorder but differs from it in that symptoms Serotonin deficiencies can give rise to wide range of
tend to outnumber signs (more subjective than objective illnesses depending on the part of the brain that is
depression) affected.
Other neurotransmitters may send the message that we
are full but serotonin level determines if we are satisfied.

• This means that


Disturbance in appetite and libido are uncharacterist ic
( in dysthymia)
Psychomotor agitation or retardation is not observed Low serotonin levels in the brain are linked to clinical
( in dysthymia) depression, suicidal tendencies, anxiety disorders,
• This all translates into depression w it h at t enuat ed obsessive compulsive behaviours (such as builimia),
symptomat ology. schizophrenia sleep problems, migraine, headaches, autism,
• In the question, the patients clincal features are not drug and alcohol addiction, Alzhiemer’s diseases and
enough to diagnose it as a major depressive disorder patterns of violent behaviour.

74 . Nihilist ic ideas are seen in :


A. Depression
B. Schizophrenia
C. OCN
D. Anxiety disorders
A
.........( AI I MS PGMEE - DEC 199 8)

• In Nihilist ic delusious the pt. feels that the world is


coming to an end there is no brain in skull, intestines
have rolled away
Dysthymia • Nihilist ic delusions are charact erist ically seen in
• Dysthymia means “ill humoured”. This term was introduced Depression.
in 1980, before that most patients were classified as having
neurotic depression. 75 . Bright light treatment has been found t o be most
• The typical features of Dysthymic disorders are feelings effect ive in treatment of:
of inadequacy, guilt, irritability and anger, withdrawl from A. Anorexia nervosa
society, loss of interest, inactivity and lack of productivity. B. Seasonal affective disorder
C. Schizophrenia
D. Obsessive compulsive disorder
B
.........( AI PGMEE - 200 6)

73 . I ncreased suicidal t endency is associated w it h:


A. Noradrenaline
B. Serotonin
C. Dopamine

HELP LI NE NO. 9 39 1 56 7 70 7
TOPI C 3: OCD PSYCHI ATRY 20

Phototherapy (bright light therapy) is used as a treatment


for seasonal affective disorder (mood disorder with
seasonal pat tern)

Biological therapy Used in


Transcranial magnetic - Resistant major depression (baseline
stimulation hypofrontality can be reversed)
- Obsessive compulsive disorder (OCD)
- Post traumatic stress disorder (PTSD)
- Map the motor cortex, help determine
hemispheric dominance & probe short term TOPI C 3 : OCD
memory
76 . Drug of choice for obsessional neurosis is
- T o improve some symptoms of Parkinson's
disease like bradykinasta, diminished reaction
A. Clomipramine
time. B. Chlorpromazine
C. Haloperidol
D. Clomiphene
A
.........(AIIMS PGMEE - DEC 1994)
AIPGMEE – 1994
AIPGMEE - 1995
AIPGMEE – 1996
AIPGMEE – 1998
AIIMS PGMEE - MAY 2007

77 . An obsessive compulsive neurosis patient is likely to


develop
Phototherapy - Seasonal depression A. Hallucination
(seasonal affective disorder)^ B. Depression
C. Delusion
- Sleep disorders D. Schizophrenia
- Decreased irritability and B
diminished functioning .........( AI I MS PGMEE - MAY - 199 3)
associated with shift work
Depression is very commonly associated with obsessive
- Jet lag compulsive disorder. It is estimated that at least half the
- OCD with seasonal variation patients of OCD have major depressive episodes.
Washers is the commonest type of obsession disorder.

78 . A person going to temple gets irresistible sense of


ideas to abuse God, w hich cannot be resisted, likely
diagnosis is:
A. Obsessive compulsive disorder
B. Mania
C. Schizophrenia
D. Delusion
A
.........( AI I MS PGMEE NOV - 199 9)

• The person is suffering from obsessive compulsive disorder


because he knows that the thoughts coming in his mind
(to abuse god) are absurd & irrational (he is trying to
Vagal - Chronic, recurrent major resist it). This shows that the thoughts are “ego alien”.
stimulation depression • This obsession or thought is followed by a behaviour which
is performed with a sense of compulsion. So the diagnosis
Depression with Sleep is O.C.N.
deprivation Ego-Alien
• Let’s consider a person who is habitually critical and
judgmental of other people... The criticalness is seen

HELP LI NE NO. 9 39 1 56 7 70 7
TOPI C 3: OCD PSYCHI ATRY 21

as “ego synt onic” for this individual ; that is, it is in Exposure and response prevent ion is the first line
accord w ith his personality and does not feel alien technique of behavioral t herapy in pat ient s w ith
or undesirable t o him . It goes along with, and is part OCN.
of, the person’s conception of himself. It is not felt as
disharmonious with his well-being.

• A trait, like criticalness, has to become “ego-alien” for a


person to do something about it, or even to see it as an
issue to be understood.
• “Ego-alien” means t hat it is experienced as being
alien to the ego, as not consistent w ith the person’s
interests, as conflicting w ith the rest of him .
Behavioral therapy with Exposure and Response prevention
79 . The drug of choice for obsessive compulsive disorder Exposure therapy Exposure techniques range from
is: - brief imaginal exposure
A. Imipramine - flooding or real life ritual evoking
B. Fluoxetine - stimulus
C. Chlopromazine Aim to decrease the anxiety associated with obsessions
D. Benzodiazepine Attempt is to ultimately decrease the discomfort associated
B with the elicit ing st imuli through habit uation
.........( AI PGMEE - 200 7)

Selective Serotonin Reuptake Inhibitors (SSRI) especially Response prevention


Fluoxetine are the drugs of choice for OCD® Involves having the patient face the feared stimuli (dirt)
Management of OCD that invokes the compulsive behavior (washing) and an
attempt is made to reduce the frequency of compulsive
Behaviour Therapy act.
- Is treatment of choice® It aims at reducing the frequency of compulsive rituals or
- Includes obsessive thoughts.
- exposure & response prevention (most
important), 82 . Abnormal thought possession is found in
- desensitization, thought stopping, flooding, A. Organic Brain syndrome
implosion therapy, and diversion B. Hysteria
conditioning C. Obsessive compulsive disorder
D. Neuroasthenia
Pharmaeotherapy C
- SSRIs are drug of choice -Fluoxetine, .........( AI PGMEE - 199 4)
fluvoxamine, paroxetine, sertraline have been approved
by FDA for OCD, Disorders of Thought includ e
Clomipramine: - Phobia
of all the tricyclic & tetra cyclic drugs, clomipramine is the - Obsessive Compulsive disorder
most selective for serotonin reuptake exceeded in this - Delusion
respect only by SSRIs. - Schizophrenia

Second line drugs include 83 . True about obsession - compulsive disorder are A/ E
Valproat e, lithium, or carbamazepine , venlafaxine, A. Ego-alien
pindolol, & MAO inhibitors esp phenelzine, buspirone, 5 B. Patient tries to resist against
hydroxy tryptamine (5-HT), L- tryptophan, & clonazepam. C. Egosyntonic
D. Insight is present
80 . A 25 year old female presents w it h 2 year hist ory of C
repetitive, irresistible thoughts of contamination w ith .........( AI PGMEE - 199 4)
dirt associat ed w it h repetit ive hand w ashing.She
r e p or t s t h e se t h ou g h t s t o b e h e r ow n a n d Obsessive Compulsive Disorder:
distressing; but is not able to overcome t hem along An idea intrudes into conscious awareness repeatedly
w ith medicat ions. She is most likely to benefit from ¯
w hich of the follow ing t herapies: Recognized as one’s own idea but is Ego alien ( foreign to
A. Exposure and response prevention. one’s personalit y)
B. Systematic desensitization. ¯
C. Assertiveness training. It is recognized to be irrational & absurd ( insight is present)
D. Sensate focusing ¯
A Patient t ries to resist against but is unable to so
..........( AI PGMEE - 200 5) ¯
Failure to resist leads to marked distress.
The patient is a case of ‘obsessive compulsive neurosis’ and is •
not responding to drag treatment.
The treatment of choice now is behavioral therapy. Differentiated by delusion as a delusion is one’s ow n idea

HELP LI NE NO. 9 39 1 56 7 70 7
TOPI C 3: OCD PSYCHI ATRY 22

but not ego alien, it is strongly believed and never personalit y disorder is likely t o exhibit all of t he
thought irrational & never resisted. follow ing feat ures, except-
• From Thought insertion - as it is not one’s own idea it is A. Perfectionism interfering w ith performance
thought to be inserted forcefully. B. Compulsive checking behaviour
C. Preoccupation with rule
85 . A 15 year old boy feels t hat the dirt has hung onto D. Indecisivensess
him w henever he passes t hrough the dirt y street. D
This repetit ive t hought causes much dist ress and .........( AI I MS PGMEE - NOV 200 4)
anxiety. He know s that there is actually no such thing
after he has cleaned once but he is not satisfied and 93 . Which of the follow ing statements different iat es t he
is compelled t o t hink so. This has led t o social obsessional idea from delusions -
w it hdraw al. He spends much of his t ime t hinking A. The idea is not a conventional belief
about the dirt and cont amina tion. This has affect ed B. The idea is held inspite of contrary evidence
his st udies also. The most likely diagnosis is : C. The idea is regarded as senseless by patient
A. Obsessive compulsive disorder D. The idea is held on inadequate ground
B. Conduct disorder C
C. Agoraphobia .........( AI I MS PGMEE - NOV 200 5)
D. Adjustment disorder
A Obsessions
.........( AI PGMEE - 200 3) • Obsessions are persistent intrusive thoughts, ideas or
impulses.
Obsession Compulsion
• An idea, impulse or image • Usually follows obsession Delusions
which intrudes into • It is aimed at either • Delusions are fixed false beliefs that are outside t he
conscious awareness preventing or neutralizing patients culture.
repeatedly It is recognised the distress or fear arising • For example a patient’s belief that his thoughts are being
as one's own idea, impulse out of obsession. • The broadcast outside his head is a delusion but a belief in
or image but is ego-ailen behaviour is not realistic santa claus is not.
(foreign to one's and is either irrational or • Because delusions are fixed false beliefs they cannot be
personality) excessive corrected by the physician. Contraindication of the
• It is recognised as • Insight is present, so the patient’s delusional belief may cause the patient to
irrational and absurd patient realises the become angry and stop t he interview .
(insight is present) • Patient irrationality or compulsion. • The physician should not pretend to agree with the
tries to resist against it, but delusion but should take a neutral position and continue
• The behaviour is
is unable to do so. • Failure the examination.
performed with a sense of
to resist leads to marked subjective compulsion
distress. 94 . Drug of choice for obsessive compulsive neurosis is:
(urge or impulse to act)
A. Fluoxetine
B. Clomipramine
88 . True stat ements about obsession :
C. Imipramine
A. It is the repetitive thoughts or images.
D. CPZ
B. The patient thought lhat the images or thoughts are
B
imposed by other’s.
.........( AI I MS PGMEE - DEC 199 7)
C. Content about sex or God
D. The patient gets disturbed when unable to remove the • Clomipramine is drug of choice for obsessive compulsive
ideas or thoughts disorders.
C&D • Fluoxetine ( SSRD is fast emerging as the 2nd drug of
..........( PGI - DEC 200 3) choice because central 5HT system is involved in OCD.
• DOC for generalized anxiety disorder Benzodiazepine
89 . True about obsessive compulsive disorder I s/ are:
A. Irresistible desire to do a thing repeatedly
B. Is a dissociative disorder
C. Denial is the defense mechanism against O.C.D
D. Patient is conscious about the disorder
E. Can cause severe distress
A, D & E
.........( PGI - June -200 1)
• DOC for Panic attack —Antidepressant (Fluoxetine)
90 . Features of obsessive- compulsive neurosis are
A. Repetitiveness
B. Irresistibility
C. Unpleasantness
D. Poor personal care
A, B & C
.........( PGI - June -200 2)

9 2. A 3 5 year old man w it h an obsessive compulsive

HELP LI NE NO. 9 39 1 56 7 70 7
TOPI C 4: MANI A PSYCHI ATRY 23

• DOC for Alzhemer’s Disease TOPI C 4 : MANI A


– Tacrine
96 . Drug of choice for prophylaxis of mania is
A. Diazepam
B. Lithium
C. Haloperidol
D. Fluoxetine
B
.........(AIIMS PGMEE - DEC 1994), AIPGMEE - 1996

Lithium is the drug of choice for both acute episodes and


prophylaxis of mania.

97 . Which drug is not used in prophylaxis of MDP :


• DOC for Nocturnal Enuresis I mipramine A. Haloperidol
B. Lithium
C. Carbamezapine
D. Valproate
A
.........(AIIMS PGMEE - MAY 1995), AIPGMEE - 1995

Established Agents for the prophylaxis of MDP


• Lit hium (1st choice)
• Carbamezapine (2nd choice)
• Valproat e
• T/T for opiod withdrawal
– Methadone (1st choice) 98. Lallu a 22 yrs. old male suffers from decreased sleep,
– Clonidine (2nd choice) increased sexual act ivity, excitement and spending
excessive money for last 8 days. The diagnosis is :
A. Confusion
B. Mania
C. Hyperactivity
D. Loss of memory
B
.........( AI I MS PGMEE MAY - 200 1)

Symptomatic profile of the patient suggests the diagnosis of


Mania
Characteristic Feature Of Mania -
• T/T for Alcohol withdrawal (1) Elation Of Mood
– Chlordiazepoxide (1st (2) Delusion of Grandeur
– Diazepam (2nd) (3) Increased psychomotor activity (usually goal oriented)
(4) Speech and thought —>
(a) More talkative
(b) Pressure of speech, use playful language
(c) Flight of ideas
(5) Reduced sleep
(6) Over activity and poor jundgement

• Symptoms of mania is DIGFAST:


– D = Distractibility
– I = Indiscretion
– G = Grandiosity
95 . Transmitt er mainly involved in OCN - – F = Flight of ideas
A. GABA – A = Activity increased
B. Serotonin – S = Sleep (decreased need for)
C. Dopamine – T = Talkativeness (pressured speech)
D. Nor Adrenaline the person is involved in high risk activities like
B – buying sprees
.........( AI I MS PGMEE - MAY 199 5) – reckless driving
– foolish business investments
• Serotonin (5 HT) is involved in Obsessive Compulsive – distributing money or articles to unknown persons
Neurosis.
99 . A 67- year old lady is brought in by her 6 children
• Neurotransmitters involved in depression ® Noradrenaline saying t hat she has gone senile. Six mont hs aft er
& Dopamine her husband’s death she has become more religious,

HELP LI NE NO. 9 39 1 56 7 70 7
TOPI C 4: MANI A PSYCHI ATRY 24

spirit ual and gives lot of money in donation. She is Carbamezapine is also efficacious.
occupied in too many activities and sleeps less. She Carbamezapine,
now believes t hat she has a goal t o change t he has clinical efficacy in the treatment of acute mania.
societ y. She does not lik e being brought t o t he
hospit al and is argumentat ive on being questioned 10 4. A 20 -year old man has present ed w ith increased
on her doings. The diagnosis is : a l coh ol con su m p t i on a n d se x u a l i n d u l g e n ce ,
A. Acute manic excitement irritability, lack of sleep, and not feeling fatigued even
B. Delusion on prolonged periods of act ivit y. All these changes
C. Schizophrenia have been present for 3 w eeks. The m ost likely
D. Depression diagnosis is:
A A. Alcohol dependence
.........( AI I MS PGMEE MAY - 200 2) B. Schizophrenia
C. Mania
10 0. All of t he follow ing conditions are included in t he D. Impulsive control disorder
diagnosis of Bipolar disorder except: C
A. Mania alone .........( AI PGMEE - 200 3)
B. Depression alone
C. Mania and depression Features of mania:
D. Mania and anxiety
- Elevated mood
B
.........( AI PGMEE - 200 7) - Increased energy and activity
Patients who are afflicted only with major depressive episodes - Flight of ideas/gradiose ideas
are said to have - major depressive disorder or unipolar - Impaired insight
depression
Patients with bot h manic and depressive episodes or - Reduced sleep early awakening
patient w ith manic episodes alone are said to have
bipolar disorder. - Increased apetite
The term unipolar mania, pure mania, or euphoric - Increased libido
mania are sometimes used for bipolar patients who do
not have depressive episodes. At least episode of one week is
required
1 01 . A period of normalsy in bet w een t w o psychotic
disorders is a feature of:
A. Schizophrenia
B. Manic depressive psychosis
C. Alcoholism
D. Depression
B
.........( AI PGMEE - 199 9)

• Bipolar mood disorders are characterized by recurrent


episodes of mania and depression at different times.
• Mania or Depression, occurs in episodes and in the
intervening period the patient in normal
• As the aged advances, the normal intervals betw een Features of Alcohol dependence:
t w o episodes shorten and the duration of episodes - Compulsive feeling to take substance
and their frequency increases. - Difficulties in controlling substnance taking behaviour
- Withdrawal state
10 2. Drug of choice for rapid cycling MDP is: - Tolerance
A. Lithium - Neglect of alternative pleasures
B. Carbamazepine - P ersistant use despite clear eviden ce of harmful
C. Sodium valproate consequences
D. Haloperidol
A Definite diagnosis of dependence is made only if at least
.........( AI PGMEE - 199 9) three of the above have been exhibited

Lithium is currently the drug of choice for treatment of


maniac episode as w ell as for prevention of further
episodes in bipolar mood disorder.
Valproic acid
an alternative in patients who cannot tolerate Lithium or
respond poorly to it.
Infact it may be better than Lithium for rapid cyclers or
who present with mixed or dysphoric mania.”

HELP LI NE NO. 9 39 1 56 7 70 7
TOPI C 4: MANI A PSYCHI ATRY 25

The patients presentation points more to a diagnosis of Mania 10 9. True about mania:
than Alcohol dependence. I n cr e a se i n a l coh ol A. Anhedonia
consumption is explained on the basis of increased B. Elated mood
impulsiveness and disinhibition that accompanies mania. C. Avolition
D. Delusion of grandiosity
10 6. Elation is cont rolled by: E. Distractability
A. Limbic system B, D & E
B. Frontal lobe .........(PGI - DEC 2002)
C. Temporal lobe PGI - JUNE 2006
D. Occipital lobe
A 11 1. Flights of idea is seen in :
.........( PGI - 1 997 - Dec) A. Mania
B. Schizophrenia
• It is believed that limbic system is closely linked with normal C. Depression
& abnormal emotional reactions. D. Delirium
• One major part of limbic system, believed to be important E. Neurosis
in emotional experiences, is papez A
circuit. This important circuit, which lies within the limbic .........( PGI - JUNE 200 3)
system, connects cingulat e bundle, hippocampus,
ant er ior t halam us, m am illar y bodies, f or nix & • Flight of ideas are seen in Mania. It is rapidly produced
sept um. speech with abrupt shifts from topic to topic using
environmental cues. Usually connections between shifts
are apparent.

11 2. True about bipolar - I I disorder :


A. Recurrent depression
B. Recurrent mania
C. Repetitive depression & mania
D. Repetitive mania & hypomania
E. Repetitive depression hypomania
E
.........( PGI - JUNE 200 5)
10 7. Mania is characterized by:
A. Paranoid delusion • Manic episodes are characterized by the following
B. Loss of orientation symptoms:
C. High self esteem - At least 1 w eek of profound mood dist urbance is
D. Loss of insight present, char acterized by elation, irr itability, or
C expansiveness.
.........( PGI - 1 999 - Dec) - Three or more of the following symptoms are present:
Grandiosity
Diminished need for sleep
Excessive talking or pressured speech
Racing thoughts or flight of ideas
Clear evidence of distractibility
Increased level of goal-focused activity at home, at work, or
sexually
¡Excessive pleasurable act ivit ies, often with painful
consequences

• Mania is characterised by high self esteem The mood disturbance is sufficient to cause impairment
• Important features of manic episodes are : at work or danger to the patient or others.
— Elevated, expansive or irritable mood The mood is not the result of substance abuse or a medical
— ted psychomotor activity . condition.
— More talkative - Grandiosity
— Goal - directed activities Hypomanic episodes are characterized by the following:
— Increased need for sleep The patient has an elevated, expansive, or irritable mood of
- Loss of insight. at least 4 days’ duration.
− Three or more of the following symptoms are present:
10 8. BPAD includes: Grandiosity or inflated self- esteem
A. Recurrent depressive episodes Diminished need for sleep
B. Recurrent manic episodes Pressured speech
C. Depressive episodes and hypomanic episodes Racing thoughts or flight of ideas
D. Manic episodes and dysthymia Clear evidence of distractibility
E. Manic episodes .and depressive episodes Psychomotor agitation at home, at work, or sexually
B& C Engaging in act ivit ies with a high potential for painful
.........( PGI - DEC 200 2) consequences

HELP LI NE NO. 9 39 1 56 7 70 7
TOPI C 5: DELUSI ON PSYCHI ATRY 26

The mood dist urbance is observable to others. C. Delusion of persecution


The mood is not the result of substance abuse or a medical D. Obsessive compulsive disorder
condition. C
.........( AI I MS PGMEE NOV - 199 9)
• Bipolar disorder constitutes one pole of a spectrum of
mood disorders including bipolar I (BPI), bipolar II (BPII), Delusion of Persecution
cyclothymia (oscillating high and low moods), and major • Pt. experiences that he is being persecuted against.
depression. • This person has feeling of passivit es ( as his brain is
• Bipolar I disorder is also referred to as classic manic- being controlled by radio w aves) but this passivites is
depression, characterized by distinct episodes of maj or a byproduct of persecution (police is behind him, he hits
depression cont rast ing vividly w it h episodes of the neighbours).
mania, w hich lead t o severe impairment of function.
• I n comparison, bipolar I I disorder is a milder disorder
consisting of depression alternat ing w it h periods of
hypomania.
• Hypomania may be thought of as a less severe form
of mania that does not include psychotic symptoms
or lead to major impairment of social or occupational
funct ion.

11 3. A person w ho laughs one minut e and cries the next


w ithout any clear st imulus is said to have - • Lets see other options :
A. Incongruent affect • Passivity Phenomenon —> Pt. experiences that his
B. Euphoria actions are under control of some external force.
C. Labile affect “He is like a Robot controlled by others.” [This pt’s passivity is
D. “Split personality” a by product of delusion of persecution]
C
..........( AI I MS PGMEE - NOV 200 5)

The emotional state that the patient experience’s internally


is known as mood and the outward expression of the
patients internal emotional state is known as affect.
Affect can be described in following ways. Appropriat e
affect
• Condition in which the emotional tone is in harmony with
the accompanying idea, thought or speech also further
described as broad or full effect in which a full range of • Thought insert ion —> Subject experiences thought
emotions is appropriately expressed. imposed by some external force on his passive mind.

Inappropriate affect
• Disharmony between the emotional feeling tone and the
idea, thought or speech accompanying it.
Blunted affect
• Disturbance in affect manifested by a severe reduction in
the intensity of externalized feeling tone.
Restricted or constricted affect
• Reduction in intensity of feeling tone less severe than
blunted affect but clearly reduced.
Flat affect
• Absence of near absence of any signs of affective
expression; voice monotonus, face immobile.

Labile affect
• Rapid and abrupt changes in emotional feeling tone 11 5. A pt . came w it h complaints of having a deformed
unrelated t o any external st imuli. The patient may nose and also complained that nobody t akes him
laugh one minute and cry the next, w ithout a clear seriously because of the deformity of his nose. He
stimulus. has visit ed several cosmetic surgeons but they have
sent him back saying that there is nothing w rong
TOPI C 5 : DELUSI ON w ith his nose. He is probably suffering from:
A. Hypochondriasis
11 4. A man hits his neighbour. Next day he feels that B. Somatization
police is behind him and his brain is being cont rolled C. Delusional disorder
by radio w aves by his neighbour. The probable D. OCD
diagnosis is : C
A. Thought insertion .........( AI I MS PGMEE NOV - 200 1)
B. Passivity feeling • The pt is suffering from delusional disorder.

HELP LI NE NO. 9 39 1 56 7 70 7
TOPI C 5: DELUSI ON PSYCHI ATRY 27

• Somatoform disorders are physical ailments (such as pain, vomiting or diarrhea


nausea, depression, dizziness) or concerns for which no • one sexual symptom such as lack of interest or erectile
adequate medical explanation has been found. dysfunct ion
• Somatoform disorders are physical symptoms that seem • one pseudoneurological symptom similar to those seen in
as if they are part of a general medical condition, however Conversion disorder such as fainting or blindness
no general medical condition, other mental disorder, or
substance is present. Hypochondriaris
• The complaints are serious enough to cause significant • in hypochondriasis
emotional distress and impairment of social and/or • patient believes that he is having some very serious
occupational functioning disorder.
• interprets some normal body function or a minor physical
• Grief that finds no vent in tears makes other organs weep abnormality (eg abdominal pain) as a very serious
• “ disorder:
—Dr. Henry Maudsley, British psychiatrist • This is not delusion as the patient may agree with the
doctor that he is exaggerating his symptoms
The somatoform disorders recognized by the Diagnostic and In the question the patient does not agree with the doctors
Statistical Manual of Mental Disorders of the American • NO I NSI GHT
Psychiatric Association are:
• Conversion disorder
• Somatization disorder
• Hypochondriasis
• Body dysmorphic disorder
• Pain disorder
• Undifferentiated somatoform disorder - only one
unexplained symptom is required for at least 6 months
• Somatoform disorder NOS

Somatoform disorders are not the result of conscious


malingering or factitious disorders.
• Hy poch on dr ia si s is often accompanied by other
psychological disorders.
• Clinical depression, obsessive-compulsive disorder phobias
and somatization disorder are the most common
a ccom p a n y i n g con d i t i on s i n p e op l e w i t h
hypochondriasis, as well as a generalized anxiety disorder
diagnosis at some point in their life.

• OCD (Obssesive compulsive disorder)


• In OCD an idea, impulse or image int rudes int o
conscious aw areness repeat edly.
• This idea is ego-alien (foreign to one’s personality)
• Somatizat ion disorder • Can be better explained by an example
• Somatizat ion disorder ( also Briquet’s disorder or, • a 21 yr: female student
in ant iquity, hysteria ) is a psychiatric diagnosis applied – suffering for the past 5 months with an irresistable urge
to patients who chronically and persistently complain to count any object she sees eg
of v a r i e d p h y si ca l sy m p t om s t h a t h a v e n o » tiles on roof
ident if iable physical origin . One common general » men on road
etiological explanation is that internal psychological conflicts
are unconsciously expressed as physical signs. • She is distressed by this preoccupation and is not able to
perform her work properly.
• Her idea is ego alien (ie the thoughts are alien to her
personality)
– she knows that what she is doing or thinking is absurd
– but she just can not help it
– (Note that the pt in question does not think herself to
be Wrong)

• Delusion-
symptoms “are fixed and culturally inappropriate but nonbizzare beliefs
recurrent and chronic that a pt holds despite all reasonable evidence to
at least 2 year duration is needed for diagnosis the contrary.”
• pt. in question has
The DSM-IV establishes the following five criteria for the • false fixed firm belief (delusion)
diagnosis of this disorder: • that her nose is deformed
• a history of somatic symptoms prior to the age of 30 • She does not agree with the doctors
• pain in at least four different sites on the body • a hypochondriac w ill agree w ith t he doct or
• two gastrointestinal problems other than pain such as

HELP LI NE NO. 9 39 1 56 7 70 7
TOPI C 5: DELUSI ON PSYCHI ATRY 28

11 6. A 25 year old university student had a fight w ith The episodes are egodystonic & the patient realize
the neighboring boy. On the next day w hile out, he the unreality of the symptoms
st arted feeling t hat tw o men in police uniform w ere
observing this movement s. When he reached home
in the evening he w as frightened. He expressed that
police w as aft er him and w ould arr est him. His
symptoms represent:
A. Delusion of persecution
B. Ideas of reference
C. Passivity
D. Thought insertion
A
.........( AI I MS PGMEE NOV - 200 3)

Delusion of Persecution:
• Depersonalization is the feeling that the body or the
The patient has delusions that he is being persecuted against,
person self is strange & unreal.
e.g. ‘people are against me1.
• Derealization is the perception of objects in the external
Thought I nsertion:
world as strange & unreal.
Subject experiences thoughts imposed by some external
force on his passive mind.
Somatic Passivity :
Bodily sensations especially sensory systems, are
experienced as imposed on body by some external force
e.g.Robot, X-ray waves

11 7. Delusion is a disorder of
A. Thought
B. Perception
C. Insight 11 9. A 30 year old unmarried w oman from a low socio-
D. Cognition economic status family believes that a rich boy staying
A in her neighbourhood is in deep love w ith her. The
.........( AI PGMEE - 200 7) boy clearly denies his love tow ards this lady. Still t he
lady insists that his denial is a secret affirmat ion of
Delusion is a disorder with disturbed content of thought. his love tow ards her. She makes desperate at tempts
Hallucination & illusion are disorders of perception to meet t he boy despite resistance from her family.
Delirium & dementia are disorders of cognition She also develops sadness at t imes w hen her effort
to meet t he boy does not materialize. She is able to
insight is disturbed in psychosis®. maint ain her daily rout ine. She how ever, remains
preoccupied w it h the thought s of t his boy. She is
11 8. A 25 -year old housew ife came t o the Psychiat ry likely to be suffering from :
out patient department ( OPD) complaining that her A. Delusional disorder
nose w as l onger t han usual. She f el t t hat her B. Depression
husband did not like her because of t he deformity C. Mania
a n d h a d d e v e l op e d r e l a t i on sh i p w i t h t h e D. Schizophrenia
neighbouring girl. Further she complained that people A
made fun of her. I t w as not possible to convince her .........( AI PGMEE - 200 4)
that t here w as no deformity. Her symptoms include:
A. Delusion This lady is suffering from erotomaniac delusional disorder
B. Depersonalization in which the content of delusion is that another person
C. Depression usually of higher status, is in love with the individual.
D. Hallucination
A
.........( AI PGMEE - 200 4)

• This lady is suffering from delusional dysmorphophobia a


kind of delusional disorder in which patient feels her/his
part of body or w hole body ugly or misshapen
And the personality is affected only in delusional area.
• Hallucination is perception in absence of external 1 2 0 . A 41 year old w oman w orking as an Execut ive
stimuli Direct or is convinced t hat t he managem ent has
• Depersonalization disorder is characterized by persistent denied her promot ion by preparing false report s
or recurrent alteration in the perception of the self to about her compet ence and have forged her signature
the extent that a person’s sense of his/her own reality is on sensit ive documents so as to convict her. She
temporarily lost. files a complaint in the police stat ion and requests
Patient may feel that they are mechanical, in a dream or for security. Despit e all t his she att ends to her w ork
detached from their bodies and manages t he household) She is suffering from :

HELP LI NE NO. 9 39 1 56 7 70 7
TOPI C 5: DELUSI ON PSYCHI ATRY 29

A. Paranoid Schizophrenia A. Othello syndrome


B. Late onset Psychosis B. Capgras syndrome
C. Persistent Delusional Disorder C. De Clerambault syndrome
D. Obsessive Compulsive Disorder D. Franklin syndrome
C C
.........( AI PGMEE - 200 4) .........( PGI - June -199 8)

Difference between schizophrenia & delusional disorder is that 12 5. Folie a deux is seen in :
in: A. Hysteria
B. Paranoid
Schizophrenia Delusional disorder C. OCD
Behaviour, Behaviour, D. Neurasthenia
Personality & Contact Personality & contact B
with reality with .........( PGI - June -199 9)
is markedly disturbed reality is disturbed in
delusional area • Folie à deux (French for “a madness shared by two”) is a
in all areas
other rare psychiatric syndrome in which a sym pt om of
wise person is psychosis ( part icularly a paranoid or delusional
normal be li ef ) is t r an sm it t ed f r om on e i nd ivi dua l t o
another.
12 1. Delusion is a disorder of: • The same syndrome shared by more than two people
A. Perception may be called folie à trois, folie à quatre, folie à famille or
B. Thought even folie à plusieurs (“madness of many”).
C. Memory
D. Judgement • This case study is taken from Enoch and Ball’s ‘Uncommon
A Psychiatric Syndromes’ (2001, p181): Margaret and her
.........( AI PGMEE - 199 9) h usb a nd M ich a el , bot h a g e d 3 4 y e a r s, w e r e
discovered to be suffering from folie à deux w hen
t h e y w e r e bot h f oun d t o b e sh a r i n g si m i l a r
12 2. A man hit s his neighbour, next day, he feels that persecut ory delusions. They believed that certain
police is behind him and his brain is being cont rolled persons were entering their house, spreading dust and
by radio w aves by his neighbour. The probable fluff and “wearing down their shoes”.
diagnosis is: • Both had, in addition, ot her symptoms supporting a
A. Personality disorder diagnosis of paranoid psychosis , which could be made
B. Passivity feeling independently in either case
C. Delusion of Persecution
D. Organic brain syndrome Various sub-classifications of folie à deux have been proposed
C to describe how the delusional belief comes to be held by
.........( AI PGMEE - 200 0) more than one person.

The man feels that he is being persecuted against by: • Folie imposée is w here a dominant person (known
- the police who is behind him. as the ‘primary’, ‘inducer’ or ‘principal’) initially forms a
- his neighbour who is controlling his brain by radio waves. delusional belief during a psychot ic episode and
imposes it on another person or persons (known as
12 3. Delusion is not present in - the ‘secondary’, ‘acceptor’ or ‘associate’) with the
A. Delirium assumption that the secondary person might not have
B. Mania become deluded if left to his or her own devices. I f the
C. Depression parties are admit ted to hospital separately , then the
D. Compulsive disorder delusions in the person w ith t he induced beliefs
D usually resolve w ithout the need of medication.
.........( AI PGMEE - 200 2) • Folie simult anée describes either the situation w here
tw o people considered to suffer independently from
Disorder Most common type of psychosis influence t he cont ent of each ot her’s
delusion delusions so they become ident ical or st rikingly
sim ilar , or one in w hich t w o people “m or bidly
• Mania Delusion of predisposed” t o delusional psychosis m ut ually
grandeur trigger sympt oms in each other
• Depression Nihlistic delusion
• Delirium Transient delusions 12 6. A 30 year old unmarried w omen of average socio-
economic background believes t hat her boss is in
• Schizophrenia Delusion of secret ly love w ith her. She rings him up at odd hours
persecution / and w rites love lett ers t o him despite his serious
reference/control/ w arnings not to do so. She holds this belief despite
infidelity/passivity contradiction from her family members and his denial.
12 4. Delusion that someone from high socio economic How ever, she is able to manage her daily activities
st atus is loving you is in: as before. She is most likely t o be suffering from-

HELP LI NE NO. 9 39 1 56 7 70 7
TOPI C 6: DEMENTI A PSYCHI ATRY 30

A. Depression disorientation to time and place, reduced level of


B. Schizophrenia attention, d rowsiness, increased o r decreased
C. Delusional disorder psychomotor activity: either apathy which can sometimes
D. No psychiatric ailment be mistaken for depression, or increased agitation.
C
.........( AI I MS PGMEE - NOV 200 4) Essential features of delirium include:
• Acute onset (hours/days) and a fluctuating course
It is a case of delusion ( Erotomanic type) • Inattention or distraction
Features of Erotomaniacal delusion • Disorganized thinking or a altered level of consciousness
These patients have delusions of secret lovers
The patients are usually woman • Causes of delirium:
The patient believes that the suitor, a person usually more D Dementia
socially prominent than herself is in love with her. E Electrolyte disorders
These patients show certain characteristics L Lung, liver, heart, kidney, brain
Generally but not exclusively women unattractive in I Infection
appearance R Rx Drugs
Usually in low level jobs and lead lonely withdrawn lives, I Injury, Pain, Stress
being single and having few sexual contacts. U Unfamiliar enviroment
They select secret lovers who are substantially higher in M Metobolic
status from them.
12 9. Characteristic finding of Subcortical dementia is :
12 7. Delusion is a disorder of: A. Memory loss
A. Thinking B. Aphasia
B. Memory C. Dyslexia
C. Perception D. Tactile agnosia
D. Intellect A
A .........( AI I MS PGMEE NOV - 199 9)
.........( AI I MS PGMEE - DEC 199 7)
Cortical Dementia Sub cortical Dementia
Cognition ( Thought) Disorders are - Site Cortex (Frontal parietal Subcortical grey matter
• Schizophrenia temporal, occipital)
• Obsessive compulsive neurosis Memor Severe, Recall helped Mild to moderate, Recall
• Delusions & phobias y loss very little by clues partially helped by clues
Motor (N) Usually • Dysarthria Dystonia
Syste • Chorea, Rigidity
Hallucinations are disorder of ®perception m
• Tremor, Ataxia
TOPI C 6 : DEMENTI A • Flexed or extended
posture
12 8. Not a feature of dementia is - Other Aphasia, Amnesia Complex delusions &
Agnosia, Acalculia depression
A. Loss of sensorium Bradyphrenia, Dyslexia
B. Wearing of dirty clothes Examp • Alzheimers disease • • AIDS • Huntingtons
C. Forget fullness le picks disease chorea • Parkinsonism •
D. Loss of neurons in brain matter Wilsons
A Aphasia, Agnosia, Dyslexia, all are features of cortical dementia.
.........( AI I MS PGMEE JUNE - 199 9)
Cortical dementias
• Loss of sensorium is a typical feature of Delirium. It is not • Alzheimer’s disease
seen in dementia. • Vascular dementia (also known as multi-infarct dementia),
• Dement ia has follow ing feat ures - including Binswanger’s disease
• Generalized loss of neurons is seen in brain matter. • Dementia with Lewy bodies (DLB)
• Impairment of intellectual functions, impairment of memory • Alcohol-Induced Persisting Dementia
(predominantly of recent memory) • Korsakoff’s syndrome
• Deterioration of personality • Wernicke’s encephalopathy
• Impairment of judgement & impulse control • Frontotemporal lobar degenerations (FTLD), including
• Impairment of abstract thinking Pick’s disease
• Impairment of all functions occuring globally. • Frontotemporal dementia (or frontal variant FTLD)
• Semantic dementia (or temporal variant FTLD)
• Progressive non-fluent aphasia
• Creutzfeldt-Jakob disease
• Dementia pugilistica
• Moyamoya disease

The brain of a person with frontotemporal dementia shows a


shrunken and sometimes asymmetric frontal lobe and a
normal parietal lobe.
Some of the hallmark signs of delirium are:
a quick onset of symptoms, disorganized thinking,

HELP LI NE NO. 9 39 1 56 7 70 7
TOPI C 6: DEMENTI A PSYCHI ATRY 31

• Substance-induced persisting dementia (related to


psychoactive use and formerly Absinthism)
• Dementia due to multiple etiologies
• Dementia due to other general medical conditions (i.e.
end stage renal failure, cardiovascular disease etc.)

• Hunt ingt on’s disease is principally a subcort ical


dementia.

• In sagittal sections, the front part of the brain of a person


with frontotemporal dementia has less perfusion, which
appears as a darker, redder color on a pseudocolor-
coded SPECT scan.

• An infarction, visible on an MRI, indicates a patient has


had a small subcortical stroke. Multiple infarctions indicate
the patient has had numerous subcortical strokes that
could lead to severe disability and cognitive decline
(dementia) in the patient.

• Creutzfeldt -Jakob disease is marked by spongiform


change in the brain.

• Brain MRI, T2 seq uence: m ult iple cor t ical a nd 13 0. A 6 5- year- old male is brought to t he outpat ient
subcortical infarct s in a vascular dementia patient clinic w ith one year illness charact erized by marked
forgetful- ness, visual hallucinations, suspiciousness,
personality decline, poor self care and progressive
deterioration in his condit ion. His Mini Ment al Stat us
Examinat ion ( MMSE) Score is 10. His most likely
diagnosis is:
A. Dementia
B. Schizophrenia
C. Mania.
D. Depression
A
..........( AI I MS PGMEE NOV - 200 2)

Let us consider each options one hy one


Subcortical dementias
• Schizophrenia -
• Dementia due to Huntington’s disease
The points which are against the diagnosis of schizophrenia
• Dementia due to Hypothyroidism
are
• Dementia due to Parkinson’s disease
1) Schizophrena has a much earlier age of onset (second
• Dementia due to Vitamin B1 deficiency
and third decades).
• Dementia due to Vitamin B12 deficiency
Schizophrenia will never present at the age of 65.
• Dementia due to Folate deficiency
2) It is associated with intact memory.
• Dementia due to Syphilis
M a r k e d f or g e t f u l n e ss i s n ot a f e a t u r e of
• Dementia due to Subdural hematoma
schizophremia.
• Dementia due to Hypercalcaemia
3) Minimental score is normal in schizophrenia.
• Dementia due to Hypoglycemia
• AIDS dementia complex
Mania -
• Pseudodementia (associated with clinical depression and
• Mania can be easily ruled out as it has radically different
bipolar disorder)
symptomatic profile.

HELP LI NE NO. 9 39 1 56 7 70 7
TOPI C 6: DEMENTI A PSYCHI ATRY 32

• MMS examination is also normal B. Disso ciation


• Depression - C. Delusions
• Easily ruled out because of radically diff . symptomatically D. Dementia
profile. D
• Sometimes severely depressed patients may show .........( AI PGMEE - 200 6)
symptoms of memory loss, a phenomenon called
pseudodem ent ia but w hen t hese pat ient s ar e 13 2. True about dement ia is A/ E
carefully assessed, memory is intact A. Often irreversible
B. Hallucination are not common
• Dementia - C. Clouding of consciousness is common
D. Nootropics have limited role
The following points favour the diagnosis of dementia.
C
1) Late onset of disease.
.........( AI PGMEE - 199 4)
2) Marked forget fulness.
3) Very low score in mini mental status examination
Clou di n g of consciou sn e ss & g r ossl y di st u r be d
4) Personality decline and poor self case. The decline in orient ation are characteristically found in delirium and
memory in a case of dementia is usually sufficient to this is
produce personality impairment. - very important point for differentiating it from dementia.
though not a regular feature, hallucinations can occur
in dementia. The patient in any question is considered to be in acute
confusional disorder with :
Minimental st atus examination - - disturbance in consciousness (altered sensorium)
• It is an easily administered test of cognitive functions. - disturbance in orient ation - to place (thinks, he is in jail)
• It is used to indicate a dementing process, provides a rough - to person (ceases to recognize the doctor and staff)
assesment of its severity and follow progression of - disturbance in perception (complains of scorpions attacking
the illness. him)
• Maximum score in a MMS-examination is 3 0.
• Scorer lower than 25 are considered abnormal and are seen 13 3. Reversible cause of dementia is
in condition such as dementia delirium etc. A. Mulli infarct
B. Senile Dementia
Neuropsychiat ry Exam Card C. Post Encephalitis
D. Hutington’s chorea
Mini-Mental Status Exam A
.........( AI PGMEE - 199 5)
Orientation
What is the -(day) (date) (month) (season) (year)? /5
Dementia
Where are yo u? (building) (floor) (cit y) (province)
Most common cause of Dementia
(country) /5 • Alzheimer’s Dementia
Regist ration • followed by Multiinfract Dementia
Name 3 objects: take 1 second to say each. Reversible cause of Dementia
Then ask patient to repeat them; give 1 point for each • 1st - Toxic Dementia (d/t drugs & toxins)
correct answer given on the first trial. • Ilnd - Hypothyroid Dementia
Repeat this until the patient can recite all 3 items. /3 • Illrd - Multi Infarct Dementia
At tent ion and calculation
Serial 7’s from 100. 1 point for each correct answer. 13 4. Cognition is:
Stop after 5 answers. Alternatively. spell “world” backward /5 A. Perception
Language B. Thought
Show the patient a watch and a pen and ask them to name C. Behavior
these items /2 D. Feeling
Repeat the following: “No ifs, ands, or buts” /1 B
Follow a 3-stage command: .........( AI PGMEE - 199 7)
Take a paper in your right hand (1pt.)
Fold it in half (1pt.) Cognition is Thought
Place it on the floor (1 pt.) /3 Conation is Action
Read and follow: “close your eyes” /1 Affect is feeling
write a sentence /1
Copy a drawing of intersecting pentagons./1
Recall
Ask for the 3 objects named above.
1 point for each correct asnwer. /3

Total Score / 30

1 3 1 . Riv ast i gm in e an d Done pe zi l ar e dr ug s used


predominantly in the management of:
A. Depression

HELP LI NE NO. 9 39 1 56 7 70 7
TOPI C 6: DEMENTI A PSYCHI ATRY 33

13 5. Cognitive disorders are :


A. Intellectualization. V ascular Dementia Alzheimer dementia
B. Depersonilization. • Onset : - Early age Late (65 yrs.)
C. Dementia group Sudden may be
D. Delirium
Less
gradual
C&D
..........( PGI - DEC 200 3) • Personality
preservation : Greater
Gradual and smoothy
• Cognitive disorders are characterized by syndromes of • Course : Stepwisc
Delirium, Dementia and Amnesia. patchy may be gradual
• Intellectualization is an ego defense mechanism, defined and smooth Good.
as excessive use of int ellect ual processes
Prognosis : Bad
( ‘logic’) to avoid affective expression ( ‘emotion’).
• Depersonalization is characterized by alterat ion in 13 7. Reversible causes of dement ia
perception or experience of self so that the feeling of A. Hypothyroidism
one’s own reality is temporarily lost or changed. It is an B. Alzheimer’s disease
‘as if ‘ phenomena. C. Vitamin BJ2 deficiency
D. Vitamin A deficiency
13 6. Vascular dement ia is characterised by :
A
A. Disorieniation
.........( PGI - JUNE 200 4)
B. Memory deficit
C. Emotional lability
D. Visual hallucination 13 8. ‘The Halstead Reitan bat tery involves all except :
E. Personality deterioration A. Finger oscillation
A, B, C & E B. Constructional praxias
.........( PGI - JUNE 200 3) C. Rhythm
D. Tactual performance
• Vascular dementia is diagnosed by following features in B
addition to general features of dementia .........( PGI - June -199 9)
- Development of multiple congitive deficits manifested by
both : • Halstead Reitan battery is a neuropsychological test
Memory impairment. batt eries.
One or more of following cognitive disturbances – It consists of 5 tests. They are :
- aphasia, apraxia, agnosia disturbances in executive 1. Category test
functioning. 2. Tactual performance test
- Significant impairment in social or occupational 3. Rhythm test
functioning. 4. Speech sound perception test
- Focal neurological signs and symptoms. 5. Finger oscillation
- Neurological deficit doesn’t occur exclusively in the
course of delirium. • Examples of neuropsychological tests include:
– the Wechsler Adult Memory Scale (WMS), the Wechsler
In vascular dementia, usually there is cardiovascular risk factors,
Adult Intelligence Scale (W AIS), and the Wechsler
hypertension, diabetes. Important features of dementia
Intelligence Scale for Children (WISC). Other tests include
:
- Impairment of int ellect ual functions, memory and the Halstead-Reitan Neuropsychological Battery , the
personality. Boston Naming Test, the Wisconsin Card Sorting Test,
- Emotional labilit y. the Benton Visual Retention Test, and the Controlled Oral
- Catastrophic reaction. Word Association
- Thought abnormalities e.g., hallucinations, delusions.
- Incontinence (fecal, urinary). Cat egory Test
- Disorient ation. • A total of 208 pictures consisting of geometric figures are
presented. For each picture, individuals are asked to decide
whether they are reminded of the number 1, 2, 3, or 4.
They press a key that corresponds to their number of
choice. If they chose correctly, a chime sounds. If they
chose incorrectly, a buzzer sounds. The pictures are
presented in seven subtests.
• The Category Test is considered the battery’s most
effective test for detecting brain damage , but does not
help determine where the problem is occurring in the
brain

Tactual Performance Test


• A form board containing t en cut -out shapes, and
Difference between Dementia of Alzheimers disease and t en w ooden block s mat ching t hose shapes are
vascular dementia placed in front of a blindfolded individual. Individuals

HELP LI NE NO. 9 39 1 56 7 70 7
TOPI C 7: ECT PSYCHI ATRY 34

are then instructed t o use only their dominant hand TOPI C 7 : ECT
to place the blocks in their appropriate space on
the form board. The same procedure is repeated using 14 1. Most common complicat ion of ECT is -
only the non-dominant hand, and then using both hands. A. Anterograde amenesia
Finally, the form board and blocks are removed, followed B. Depression
by the blindfold. From memory, the individual is asked to C. Psychoses
draw the form board and the shapes in their proper D. Retrograde Amnesia
locations. The test usually takes anywhere from 15 to 50
D
minutes to complete. There is a time limit of 15 minutes
.........( AI I MS PGMEE - SEP 199 6)
for each trial, or each performance segment.

Trail Making Test Adverse effect of ECT


• This test consists of two parts. Part A is a page w ith
25 numbered circles randomly arranged . Individuals In modified ECT both Retro & Antegrade amnesia is found
are instructed to draw lines between the circles in but Retrograde Amnesia is more common.
increasing sequential order until they reach the circle
labeled “End.” Part B is a page with circles containing the Ot her side effects are -
letters A through L and 13 numbered circles intermixed ( 1) Side effects associated with general Anaesthesia
and randomly arranged. Individuals are instructed to (2) Confusion in the post ictal period may occur
connect the circles by drawing lines alternating between (3) Other side effects Headache, prolonged apneas,
numbers and letters in sequential order, until they reach prolonged seizure, cardiovascular dysfunction, emergent
the circle labeled “End.” If individuals make mistakes, the mania, muscle aches & apprehension
mistakes are quickly brought to their attention, and
continue from the last correct circle. The test takes
14 2. Electro convulsive therapy is useful in :
approximately five to 10 minutes to complete.
A. Mania
Finger Tapping Test
B. Chronic schizophrenia
• Individuals place their dominant hand palm down, fingers C. Acute depression
extended, w it h t he index finger resting on a lever D. Panic disorders
that is attached to a count ing device. I ndividuals C
are instructed to tap their index finger as quickly .........(AIIMS PGMEE - FEB - 1997)
as possible for ten seconds, keeping the hand and
arm st ationary. This trial is repeated five to 10 times, • T/T for Acute Depression — E.C.T.
until the examiner has collected counts for five consecutive Chronic conditions generally do not respond to ECT
trials that are within five taps of each other. Before starting
the test, individuals are given a practice session. They are
also given brief rests between each 10-second trial, and
one to two-minute rests after every third trial. This entire
procedure is repeated with the non-dominant hand. The
test takes approximately ten minutes to complete.

Speech Sounds Percept ion Test


• Sixty tape-recorded nonsense syllables containing the
sound “ee” (for example, “meer” and “weem”) are
presented.
– After each syllable , individuals underline, from a set of
four written syllables, the spelling that represents the
syllable they heard • I ndications of ECT
1) Major severe depression with suicidal risk –
13 9. Treatable causes of dementia are: (First & most important indication)
A. Alzheimer’s ds 2) Acute schizophrenia (Least effective in chronic)
B. Hypothyroidism 3) Acute mania (ECT in Mania & Schizophrenia is not the t/t
C. Multi-infarct dementia of choice)
D. SDH (subdural h’ge) 4) Catatonic schizophrenia
E. Hydrocephalus 5) Acute psychosis
B, D & E 6) Delusional depression
.........( PGI - June -200 1)
Contraindications of ECT
14 0. Biological Amnesia is -
• Raised ICT (Most important)
A. Presenile dementia
B. Lack of interest • Myocardial infarction
C. Opiod drug addiction • Hypertension
D. Hypothyroidism • CVA
A • Retinal detachment
.........( AI I MS PGMEE - DEC 199 8)

HELP LI NE NO. 9 39 1 56 7 70 7
TOPI C 7: ECT PSYCHI ATRY 35

14 7. I ndications for ECT are :


A. Paranoid schizophrenia
B. Depression with suicidal tendency
C. Catatonic schizophrenia
D. Neurotic depression
E. Depression in involutional stage
B& C
.........(PGI - 2000 - Dec)
PGI - JUNE 1997
PGI - June -1998
14 3. Maximum benefit of elect roconvulsive t herapy is in
A. Hysteria • First and most important indication of ECT is major severe
B. Chronic schizophrenia depression w ith suicidal tendencies
C. Mania • Other indications
D. Depression with suicidal tendency — Severe catatonia
D
— Severe psychosis (Schizophrenia or Mania)
.........( AI I MS PGMEE - NOV - 199 3)

14 4. A patient comes in stupor condit ion parent s give Indication of ECT in depression (Major)
history of being continually sad & suicidal att empt s, • Suicidal tendencies
not eating and sleeping most of the t ime. T/ T is : • Poor intake of food and fluids
A. ECT • With psychotic feature
B. Antidepressant • With melanocholia
C. Antipsychotic • Unsatisfactory response to drugs
D. Sedatives • When speedier recovery needed.
A
.........( AI I MS PGMEE JUNE - 200 0) 14 9. Absolute contraindicat ion for ECT :
A. Increased ICT
The pt. is suffering from severe depression B. Pregnancy
• Insomnia
C. Recent MI
• Not eating
A
• Continuous in sad mood
• Suicidal attempts .........( PGI - JUNE 200 6)
• ECT is the t/ t of choice for severe depression
Absolute contraindications for ECT
- Raised intracranial tension
• Relative contraindications:
- Recent MI
- Severe hypertension
- CVA
Severe pulmonary disease
- Retinal detachment
- Pheochromocytoma
- Pregnancy is not a cont raindication t o ECT.
• Indications
• ECT is used predominantly as a treatment for depression.
15 1. ECT in depressive phase of MDP is useful because it
– It is generally reserved for use as a second-line treatment
:
for patients who have not responded to drugs. The first-
line use of treatment is for situations where immediate A. Produces recurrence
clinical intervention is needed or alternative treatments B. Reduces recurrence
are not advisable. C. Shortens duration
D. Increases drug effects
ECT is also sometimes used in the treatment of other disorders, C
for example, schizophrenia, mania, and cat atonia .........( PGI - June -199 9)

14 5. Rathi a 26 yr old femals diagnosed to be suffering ECT in depresssive phase of MDP is useful because it shortens
from depression. Now for the past 2 days had suicidal duration of depressive episode.
tendencies, t houghts and ideas the best T/ t is:
A. Selegiline 15 2. ECT is indicat ed in
B. Amitryptiline A. Delusional depression
C. Haloperidol + Chlorpromazine
B. Schizophrenia
D. ECT
C. Mania
D
.........( AI I MS PGMEE MAY - 20 01) , AI PGMEE - 199 4) D. Neurotic depression
A
• In suicidal depression ECT is t/f of choice. .........( AI I MS PGMEE - JUNE 199 8)

HELP LI NE NO. 9 39 1 56 7 70 7
TOPI C 8: FLUOXETI NE PSYCHI ATRY 36

• In Delusional depression, Nihillistic delusions may lead to • Fluvoxamine


suicide so persistant delusions in depression is indication • Paroxetine
of ECT.
• Neurot ic Depression is a mild atypical depression. • Mechanism of action of atypical TCAs
• It is not even severe enough to be called depressive • Tianeptine
episode and last s 2 yrs. or more, so no question of • It increase rather than inhibit 5-HT uptake
ECT. • Mianserin
T/T of choice is à Psychot herapy • does not inhibit either NA or 5-HT uptake, it blocks
presynaptic alpha-2 receptors, increases release and
• ECT is not indicted in Schizophremia and mania. turnover of NA in brain
• But it may be indicted in some special conditions of
shizophrenia — > • Tianeptine is described as a selective serot onin reuptake
• Severe schizophrenia not responding to drugs enhancer ( SSRE) , structurally similar to the tricyclic
• Catatonic stupor antidepressants.
• Unlike the tricyclics, however, it enhances the reuptake
• Catatonic excitement
of serotonin inst ead of blocking it.
• Severe psychosis not responding to drugs.
• Interestingly, tianeptine along with its two metabolites
does not affect uptake of monoamines (i.e. DA, 5-HT,
TOPI C 8 : FLUOXETI NE
and noradrenaline) in vitro.

15 3. Mode of act ion of Fluoxetine is - • Tianeptine show s efficacy against serious depressive
A. GABA inhibition e p i sod e s ( m a j or d e p r e ssi on ) , compar able to
B. Adrenergic neuron blocking action amitriptyline, imipramine and fluoxetine, but with fewer
C. Inhibit Axonal uptake of 5HT side effects.
D. Alpha Adrenergic stimulation • It was shown to be more effective t han maprot iline
C in a group of patients w ith co-existing depression
.........(AIIMS PGMEE - SEP 1996) and anxiet y.
AIPGMEE - 1999
• Tianeptine also displays significant anxiolytic properties and
• Fluoxetine is a tricyclic antidepressent (like imipramine, is useful in treating a spectrum of anxiety disorders including
Amitriptyline) panic disorder,
• Tianeptine is claimed to have strong antidepressant
• While typically TCA inhibit uptake of both NA and 5 HT and anxiolyt ic propert ies with a relat ive lack of
by neurons. sedative, anticholinergic and cardiovascular adverse
effect s, thus suggesting it is particularly suitable for
• Fluoxetine is select ive serotonin reuptake inhibit or use in elderly patients and in those follow ing alcohol
( SSRI ) w ithdraw al; such patients can be more sensitive to the
• Therefore it is devoid of follow ing side effects adverse effects of psychotropic drugs.
(1) Anticholinergic
(2) Sedation
(3) Hypotension

• Fluoxetine hydrochloride is an antidepressant of the


select ive serotonin reuptake inhibit or ( SSRI ) class.
• Fluoxetine is approved for the treatment of depression
( i n cl u d i n g p e d i at r i c de p r e ssi on ) , ob se ssi v e - Side effects are as follows (Amitriptyline vs Tianeptine):
compulsive disorder ( in both adult and pediatric • dry mouth (38 vs 20%)
populations) , bulimia nervosa, panic disorder and • constipation (19 vs 15%)
premenstrual dysphoric disorder. • dizziness/syncope (23 vs 13%)
• Other indications include hypochondriasis and body • drowsiness (17 vs 10%)
dysmorphic disorder. • postural hypotension (8 vs 3%)
• I nsom nia and night mares occur mor e of t en in
• Ot her SSRI s tianeptine than in amitriptyline recipients (7 vs 20%)

HELP LI NE NO. 9 39 1 56 7 70 7
TOPI C 8: FLUOXETI NE PSYCHI ATRY 37

Coping with suicide risks Penfluridol and clozapine are antipsychotics & Buspirone
• As is gener a lly true for activ ating /nonsedating is an anxiolytic.
antidepressants, par t icularly agit at ed pat ient s or
those developing increase of energy t ogether w ith 15 6. Drug of choice for panic disorders is
suicidal t hought s bef or e r em ission occur s w ill A. Fluoxetine
normally need initial comedication (1 to 4 weeks) w ith B. Lithium
a n e f f e ct i v e se d a t i n g d r u g su ch a s a C. Diazepam
benzodiazepine, barbiturate or neuroleptic. D. Chlorpromazine
Additionally, hospitalisation of these patients is desirable (close A
observation possible). These measures to lower the risk .........( AI I MS PGMEE - MAY - 199 4)
of suicide should be continued until remission of depression
is stable. • T/T of Panick attack -
Urgent t/t
• Mianserin is a tetr acyclic antidepressant that has Sublingual dose of alprazolam or lorazepam.
antihistaminic and hypnosedative, but almost no Sustained t/t
anticholinergic, effect . SSRI’s are the initial drug of choice.
• Mianserin is a w eak inhibit or of nor epinephr ine TCA’s and MAO inhibitors also have similar efficacy but they
reupt ake and st rongly st imulat es t he release of have several adverse effects.
norepinephrine.
• Interactions with serotonin receptors in the central 15 7. Drug of choice for obsessive compulsive disorders
nervous system have also been found. Its effect is usually A. Imipramine
noticeable after one to three weeks. B. Fluoxetine
• It has been phased out in favor of Mirtazapine. C. Nortriptyline
D. Trimipramine
• Mianserin blocks inhibit ory á2 - aut orecept or s on B
cent ral noradrenergic nerve endings, and so may .........( AI I MS PGMEE - MAY - 199 4)
increase the amount of noradrenaline in the synaptic cleft.
It may also cause agr anulocyt osis and aplast ic • Drug of choice for obsessive compulsive neurosis is either
anaemia. SSRI’s or clomipramine.
• Among TCA’s only clomipramine is effective in the t/t of
15 4. Mode of action of Fluoxetine is- obsessive compulsive neurosis
A. GABA inhibition • SSRI’s have similar efficacy as clomipramine without its
B. Adrenergic neuron blocking agent adverse effects, so it is fast emerging as an alternative to
C. Inhibition axonal uptake of 5HT clomipramine
D. Alpha adrenergic stimulation
C 15 8. Follow ing drugs have abuse liabilit y except:
.........( AI I MS PGMEE - JUNE - 199 7) A. Buprenorphine
B. Alprozolam
• Fluox et ine is SSRI (Selective serotonin reuptake C. Fluoxetine
inhibitor) D. Dextropropoxyphene
C
Other SSRIs .........( AI PGMEE - 199 9)
• Fluvoxamine
• Paroxetine Fluoxetine is a prototype of the newer SSRI I.e. Selective
• Also remember mechanism of action of these atypical TCAs Serotonin reuptake Inhibitors.
• Tianeptine - It increase rather than inhibit 5-HT uptake It is devoid of the common side effects, a seen with tricyclics
• Mianserin - does not inhibit either NA or 5-HT uptake, it i.e. It has :
blocks presynaptic alpha-2 receptors, increases relaease 1. No anti-cholenergic side effects.
and turnover of NA in brain 2. No sedation.
3. No hypertensive side effect.
15 5. The NEWEST antldepressant I s
A. Buspirone Fluoxetine has a newer constellation of side effects which
B. Fluoxetine include:
C. Penfluridol Nevousness , Anxiet y , I nsomnia, Nausea, diarrhea
D. Clozapine and Headache.
B Buprenorphine (opoid), alprazolam (anti-anxiety as well as
.........( AI I MS PGMEE - MAY - 199 3) anti-depressant) and dextro-propoxyphene (opoid), all are
know n to have sedat ive properties.
The new er antidepressants are -
1) Selective serotonin reuptake inhibitors - 16 0. Side effect s of fluoxetine are A/ E
• Fluoxetine A. Weight gain
• Venlafaxine B. Sweating
• Fluovaxamine C. Urinary retention
• Nefazodone D. Diarrhoea
• Paroxet ine A
• Mirtazapine .........( AI PGMEE - 199 4)

HELP LI NE NO. 9 39 1 56 7 70 7
TOPI C 8: FLUOXETI NE PSYCHI ATRY 38

• Among the common adverse effects associated with – It also has special neurologic indications for Gilles de la
fluoxetine and listed in the prescribing information, the Tourette syndrome and resistant tics.
effects with the greatest difference from placebo are – The side effects include akathisia, tardive dyskinesia,
nausea ( 22% vs 9% for placebo), insomnia (19% vs neur ole pt ic m alignant syndr om e and long QT
10% for placebo), somnolence (12% vs 5% for placebo), syndrome.
anorexia (10% vs 3% for placebo), anxiety (12% vs 6%
for placebo), nervousness (13% vs 8% for placebo), • Trazodone is a psychoactive compound with sedative,
asthenia (11% vs 6% for placebo) and tremor (9% vs anxiolytic, and ant idepressant properties .
2% for placebo). Those that most often resulted in • antidepressant activity becomes active in the first week
interruption of the treatment were anxiet y, insomnia, of therapy.
and nervousness ( 1 - 2 % each) , and in pediat ric • Trazodone has less prominent anticholinergic (dry
trials—mania (2%).[ mouth, constipation, tachycardia) and adr enolyt ic
• Similarly to other SSRIs, sexual side effects are common (hypotension, male sexual problems) side effects than most
with fluoxetine; they include anorgasmia and reduced tricyclic antidepressants.
libido. • it b e l on g s t o t h e f a m i l y of t e t r a cy cl i c
ant idepressants.
• In addition, rash or urticaria, sometimes serious, was
observed in 7% patients in clinical trials; one-third of these • Trazodone is a serotonin reuptake inhibitor and is also
cases resulted in discontinuation of the treatment. a 5- HT2 receptor antagonist.
Postmarketing reports note several cases of complications • However, in cont rast t o t he select ive ser ot onin
developed in patients with rash. The symptoms included reuptake inhibit ors such as fluoxetine , trazodone’s
vasculitis and lupus-like syndrome . Death has been antidepressant effects may be due to its antagonistic
reported to occur in association with these systemic effect at the 5-HT2 receptor site
events.[11]
Trazodone Uses
• Akathisia, that is inner tension, rest lessness, and • Clinical depression with or without anxiety
t he inabilit y to st ay st ill, oft en accompanied by • Chronic insomnia
“constant pacing, purposeless movements of the • Fibromyalgia, to control sleeping.
feet and legs, and marked anxiety,” is a common side • Control of nightmares or other disturbed sleep
effect of fluoxetine • A sleep aid (with a reduced risk of dependency)
• Endocrine
16 1. Drug of choke in depression in old person is • Decrease and, more rarely, increase in libido, weight gain
A. Fluoxetine and loss, and rarely, menstrual irregularities, retrograde
B. Buspirone ejaculation and inhibition of ejaculation.
C. Amitryptyline • Elevated prolactin concentrations have been observed in
D. Imipramine patients taking trazodone
A
.........( AI PGMEE - 199 6) • Trazodone has been associated with the occurrence of
priapism.
Fluoxetine is devoid of Ant icholinergic, Hypotensive &
sedative side effects so safer in elderly w ith cardiac – In approximately 33% of the cases reported, surgical
disease & BPH. intervention was required and, in a portion of these cases,
permanent impairment of erectile function or impotence
1 6 2. All of t he follow ing agent s are ant idepressant s resulted. M a l e p a t i e n t s w i t h p r ol on g e d or
except : i n a p p r op r i a t e e r e ct i on s sh ou l d i m m e d i a t e l y
A. Trazodone discontinue the drug and consult their physician. If the
B. Amitriptyline condition persists for more than 24 hours, it would be
C. Fluoxitine advisable for the treating physician to consult a urologist
D. Pimozide or appropriate specialist in order to decide on a
D management approach
.........( AI PGMEE - 199 7)
16 3. HI AA is a metabolite of:
Pimozide A. Serotonin
is classified amongst Anti-Psychotics. B. Dopamine
Trazodone: atypical anti-depressant C. Epinophrine
Fluoxetine : Selective serotonium reuptake Inhibitor (SSRI) D. Histamine
Amitryptiline : Tricyclic Anti-depressants A
.........( AI I MS PGMEE - MAY 200 6)
• Pimozide is an antipsychotic drug
– It has a high potency compared t o chlorpromazine • Serotonin or 5 hydroxyt rypt amine is catabolized to
(ratio 50-70:1). 5 Hydroxy indole acetic acid ( 5 HI AA) .
– On a weight basis it is e ve n m or e p ot e nt t ha n • The enzyme which catalyzes this reaction is → Monoamine
haloperidol. oxidase.
– As it has severe side effects, it is considered a drug of • 5 HIAA is excreted normally in urine. Normal adult excretes
last resort , typically prescribed only after the patient about 7 mg of HIAA per day.
has failed to respond to other medications.

HELP LI NE NO. 9 39 1 56 7 70 7
TOPI C 9: HALLUCI NATI ON PSYCHI ATRY 39

TOPI C 9 : HALLUCI NATI ON A) Essential symptoms -


a) Cognitive and intecllectual impairment -
16 4. Perception w ithout stimuli is - The impairment involves functions shuch as
A. Delusion Thinking
B. Delirium • Memory
C. Illusion • Perception
D. Hallucination • Attention
D • This impairment may be global involving almost all functions
.........( AI I MS PGMEE - SEP 199 6) or circumscribed i.e., involving some
functions.
Hallucination →perception without stimuli
b) Al t e r e d e m ot i on a l i t y - The common emotional
Symptoms seen are -
Apathy
Irritability
Euphoria
Depression
anxiety and elation may be present

c) D i st u r b a n ce o f v i g i l a n ce - The pati ent shows


disturbance of vigilance and consciouness. This usually
accompanied by perceptual abnormalities such as
I llusion → Misinterpretation of stimuli hallucinat ion and illusion.

Or ganic ment al disorder s vs non or ganic m ent al


disorders

Features Organic Non Organic


• Disturbance of May be present Absent
consciousness
• Disturbance of May be present Absent
orientation
• Disturbance of May be present Absent
memory
• Disturbance of May be present Absent
Delusion → Fixed firm belief not in keeping with social intelligence
A. educational background. • History suggestive of May be present Absent
organic
aetiology e.g.
Headache
Vomiting
Fever
Convulsions
Altered sensorium
Incontinence
• Compensatory dificits Usually underplayed by If present it is usually
patient exaggerated by the
patient
• Hallucination If present all sensory If present - mainly
sensory modalities auditory

1 6 6 . False sense of percept ion w it hout any ext ernal


16 5. The charact eristic sympt om of organic psychosis is object or st imulus is know n as:
A. Hallucination A. Illusions
B. Depression
B. Impulse
C. Transient delusion C. Hallucination
D. Anxiety
D. Phobia
A C
.........( AI I MS PGMEE - MAY - 199 3) .........( AI I MS PGMEE MAY - 200 3)
Organic mental disorders have been classified into three Hallucination →A perception that occurs in the absence
main categories • of a stimulus
1) Delirium, Dement ia and amnestic and other cognitive
I llusions → A misinterpretation of stimuli arising from
disorders external objects.
2) Mental disorders due to general medical condition
Phobia → An irrational fear of an object, situation or activity
3) Substance related disorders. I mpulse → A sudden often unreasoning determination to
• The following symptoms provide presumptive evidence perform some act.
for the existence of brain dysfunction.

HELP LI NE NO. 9 39 1 56 7 70 7
TOPI C 9: HALLUCI NATI ON PSYCHI ATRY 40

16 7. Hallucination is a disorder of: 17 0. The follow ing is suggest ive of an organic cause of
A. Perception the behavioural symptoms:
B. Thought A. Formal thought disorder
C. Intellegence B. Auditory hallucinations
D. Memory C. Delusion of guilt
A D. Prominent visual hallucination
.........( AI PGMEE - 199 9) D
.........( AI PGMEE - 200 2)
Disorders of perception include :
• Hallucinat ion : a perceptionthat occurs in absence of a • Prominent visual hallucinat ion
stimulus. Or g an ic m e nt a l di sor de r s have demonstr able and
• I llusions : a mis-interpretationof stimuli arising from external independent diagnosable cerebral disease and highly
objects. suspected when there is presence of
• Depersonalization and Derealization : are disorders in the - Visual or other non auditory hallucinations.
perception of a person’s realit y. - Soft neurological signs, Confusion, Disorientation, Memory
• Somat ic passivit y phenomenon\ : Is the presence of impairment,
sensation describe by the patient as being imposed n the
body by ‘some external agency’ , w ith t he patient being
a passive recipient.

168 . Hallucinations are:


A.Feeling of familiarity with unfamiliar things
B.Alteration of perception of ones reality
C.Mis-interpretation of stimuli
D.Perception occuring without external stimulation
D
.........( AI PGMEE - 199 9)

• Hallucinat ion: a perception that occurs in absence of


stimulus.
• D e j a v u : A sense of familia rity with unfamiliar
circumstances.
• Depersonalisation : An alteration in perception of self,
so that feeling of one’s reality is temporarily lost. - Neurological signs & symptoms as seizures, impairment of
• I llusion : misinterpretation of stimuli arising from external consciousness, sensory or motor disturbance.
object. - H/O Head injury, Drug or alcohol use
- Concurrent medical or neurological illness
16 9. All of t he follow ing are feat ures of hallucinations, - 1st episode, sudden onset, older age of onset.
EXCEPT:
A. It depends on will of the observer Most common Non organic hallucinat ions -Auditory
B. Occurs in inner subjective space Most common Organic Hallucinat ions -Visual
C. It is a vivid sensory perception
D. It occurs in the absence of perceptual stimulus 1 7 1 . Al l of t h e f ol l ow i n g a r e f e a t u r e s of
A hallucinations,except:
.........( AI PGMEE - 200 2) A. It is independent of the will of the observer
B. Sensory organs are not involved
Hallucinations are sensory perceptions, which occur in the C. It is as vivid as that in a true sense perception
absence of external stimuli. D. It occurs in the absence of a perceptual stimulus
Hypnagogic hallucinat ions:
these are vivid perceptions, which occur at the onset of ..........( AI PGMEE - 200 3)
sleep and
are associated with fearfulness. Hallucination
It is involuntary • It is perception w ith out stimuli
• I t is independent of w ill of the observer
• It occurs in inner subjective space ie sensory organs
are not involved
• It is a vivid sensory perception but not very w ell defined
as a t rue sense percept ion and imagery.

17 2. False preception w ithout external st imulus is


A. Illusion
B. Hallucination
C. Delirium
D. Delusion
B
.........( AI PGMEE - 199 8)

HELP LI NE NO. 9 39 1 56 7 70 7
TOPI C 10: PHOBI A PSYCHI ATRY 41

Hallucination : is a perception that occurs in the absence of C. Anxiety neurosis


stimulus C. Mania
Illusion : is misinterpretation of stimuli arising from external D. Depression
objects. A
Delusion: is a false unshakable belief, which is not amenable .........( AI I MS PGMEE - FEB - 199 7)
to reasoning and is not in keeping with the patients, socio
cultural and educational background. • Systemic Desensitization is t/t of choice in phobias and
Delerium : is an acute confusional state characterised by obsessive compulsive Diorders.
alt erat ion in consciousness or ient at ion, and
perception.

17 3. A patient seen a rope and fears like a snake. I t is


called:
A. Illusion
B. Hallucination
C. Delusion
D. Depresonalization
E. Deprealizatiou
A
.........( PGI - DEC 200 2) • Ot her Modalities in t/ t of phobias
(1) Psychotherapy →Not very successful
174. Formed visual hallucinations are seen I n w hich lesion (2) Drug T/ T
: • Benzodiazepines (Alprazolam) is DOC when drug t/t is
A. Frontal indicated
B. Occipital • Antidepressants → Imipramine & phenelzine
C. Temporal • Recently SSRI S (Fluoxetine & Sertraline) have been found
D. Parietal to be very effective in t/t of phobias.
C
.........( PGI - June -200 0)

• Formed visual hallucinations are seen in Temporal lobe lesion.


• Lesions of Temporal lobe (dominant side) causes:
— Dyphasia, Dyslexia, poor memory
— Complex hallucinations (smell, sound, vision)
— Homonymous hemianopia
Non-dominant temporal side lesion causes :
— Poor non-verbal-memory
— Homonymous hemianopia
— Loss of musical skills
— Complex hallucinations

17 5. Perceptual misinterpretat ion of a real obj ect is


A. Illusion 1 7 7 . A person has poor performance in front of his
B. Delusion seniors in public place, he has tachycardia w hen he
C. Hallucination has to deliever a lecture, avoids going t o parties t he
D. Schizophrenia diagnosis is:
A A. Panic disorder
.........( AI I MS PGMEE - NOV 200 7) B. Social phobia
C. Schizophrenia
• Both illusion and hallucination are perceptual disorders. D. Avoidant personality
• In illusion there is misint erpret at ion of t he act ual B
st imulus whereas in hallucination perception occurs in .........( AI I MS PGMEE JUNE - 199 9)
the absence of any stimulus.
• I llusion → Misinterpretation of rope as snake. • Social Phobia is characterized by irrational fear of performing
• Hallucination → Able to see snake without any rope activities in presence of other people or interacting with
Delusion other, the pt. is afraid of his actions being viewed by others
• It is the disturbance in the content of thought. critically resulting in embarrasment.
• It is a belief based on incorrect inference about external
reality and not consistent with patient’s intelligence and
sociocultural background.

TOPI C 10 : PHOBI A

17 6. Desensit ization form of behaviour therapy is used


for:
A. Phobic disorder

HELP LI NE NO. 9 39 1 56 7 70 7
TOPI C 10: PHOBI A PSYCHI ATRY 42

• Avoidant Personality Disorder : ? C. Depression


• It is very difficult to distiguish b/w Avoidant personality D. Organic brain syndrome
disorder & Social phobia because people with Avoidant A
personality disorder often have social phobia as w ell. .........( AI PGMEE - 200 0)

Systemic desensitization is the treatment of choice for two


important conditions :
a. phobia
b. Obsessive – compulsive neurosis

17 9. Treatment of choice for phobic disorder is


A. Behaviour therapy
B. Benzodiazepines
C. Psychotherapy
• An avoidant personality disorder is shy, has few friends, D. 5-HTreuptake inhibitors
mostly spends his time alone, does not part icipate in A
any social act ivit y . He may have social phonia as well. .........( AI PGMEE - 199 7)

The treatment of choice for Phobic disorder is Behaviour


therapy, which is usually successful.
In resistant cases Alprazolam, or Anti-depressants may be of
help.
Psychotherapy is not usually helpful

18 0. Phobia is:
A. Psychosis
B. Fear of animal
C. Anxiety
D. Neurosis
• But a social phobic guy (as the pt in question) is socially D
normal, has friends spends his time cheerfully until he is .........( AI PGMEE - 199 8)
faced w ith a sit uation w here he has t o speak in
front of an audience. • Neurosis, also know n as psychoneurosis or neurotic
disorder, is a “catch all” term that refers to any mental
imbalance that causes distress, but, unlike a psychosis
or some personality disorders, does not prevent or
affect rational thought.

Phobia is defined as an irrational fear of a specific object,


situation or activity, often leading to p e r sist e n t
avoidance of t he feared object , sit uation or activity.

Phobia is a neurosis because :


• Symptoms cause subjective distress to the patient
• I nsight is present
• Contact with realit y is preserved
• Personalit y and behaviour are relatively preserved
• There is an absence of organic causative factors.
• In avoidant personality disorder there is a persistant feeling
of tension and apprehension.

17 8. Syst emic de- sensitizat ion therapy is used for:


A. Phobia
B. Schizophrenia

HELP LI NE NO. 9 39 1 56 7 70 7
TOPI C 10: PHOBI A PSYCHI ATRY 43

18 1. Agarophobia is: 18 4. A middle aged person reported to Psychiatric OPD


A. Fear of open spaces w ith t he complaints of the fear of leaving home, fear
B. Fear of closed speces of travelling alone and fear of being in a crow d. He
C. Fear of heights develops m ar k ed anx iet y w it h palpit at ions and
D. Fear of crowded places sw elling if he is in these situations. He oft en avoids
A& D public transport to go to his place of w ork. The most
.........( PGI - 1 997 - Dec) likely diagnosis is:
A. Generalised anxiety disorder
• Agoraphobia is an irrational fear of situations. It is the B. Schizophrenia
commonest type encountered in clinical practice. It is C. Personality disorder
characterized by an irrational fear of being in places away D. Agoraphobia
from the familiar setting of home. Although it was earlier D
thought to be a fear of open spaces only, now it includes .........( AI I MS PGMEE - MAY 200 6)
fear of open space, public places, crow ded places,
any other place from where there is no easy escape to a
safe place.
• Examples of other simple phobias:
Agarophobia = Fear of high places
Zoophobia = Fear of animals
Xenophobia = Fear of strangers
Algophobia = Fear of pain
Claust rophobia = Fear of closed places.

18 2. Features of agarophobia :
A. Irritability
B. Phobic anxiety
C. Avoidance 18 5. A 50 year old male feels uncomfortable in using
D. Multiple phobia lift, being in crow ded places and t ravelling. The most
A, B & C appropriate line of treat ment is -
.........( PGI - JUNE 200 6) A. Counselling
B. Relaxation therapy
Agarophobia means fear of open space. Presently it also C. Exposure
includes its related aspects e.g. presence of crowds and D. Covert sensitization
the difficulty of immediate easy escape to a safe space C
(usually home). Most cases are women and its onset in .........( AI I MS PGMEE - NOV 200 5)
early in adult life.
This patient is suffering from Agoraphobia
Treatment
1) Psychological t / t
• Exposure t / t
Graded exposure and flooding can reduce the fear of being
in crowded places.
• Cognit ive behaviour t herapy
Cognitive therapy is about as effective as medication
probably more effective in long term.

2) Medications
The drug t/t of Agoraphobia resembles panic disorder
(a) Anxiolytic drugs
Diagnostic guidelines (WHO) :
(b) Antidepressant drugs (Tricyclic)
a) The psychological or autonomic symptoms must be
(c) Selective serotonin reuptake inhibitors
primarily manifestation of a anxiety and not secondary to
(d) Monoamine oxiduse inhibitors
other symptoms such as - delusions or obsessional
thoughts.
“I n ear l y cases t h e pat i ent s shoul d be st r on gly
b) The anxiety must be restricted to or (occur mainly in ) at
encourged t o r et urn t o t he sit uat ions t hey are
least two of the following situations: Crowds, public places,
avoiding. The t/t of choice for established case is a
travelling away from the home and travelling alone, and
combination of exposure to phobic situations with trainine
c) Avoidance of phobic situation must be, or have been a
in copins with panic attacks”.
prominent feature.
More on exposure therapy
18 3. Definitive treatment of all t ypes of phobias:
Exposure can be carried out in two main ways
A. Behavioral therapy
(i) in practice, that is in the actual situations that provoke
B. Social therapy
anxiety.
C. Avoidance
(ii) in im aginat ion, that is while imagining the phobic
D. Drug therapy
situations vividly enough to induce anxiety. I n both these
A
procedures exposure can be done in t he follow ing
.........( PGI - JUNE 200 5)

HELP LI NE NO. 9 39 1 56 7 70 7
TOPI C 11: ALCOHOLI SM PSYCHI ATRY 44

t hree w ays depending upon t he int ensit y of the – Persistence use of substance despite clear evidence of
exposed stimuli. harmful consequence
( a) Desensit izat ion - – Tolerance
Exposure can be gradual, starting with situations that provoke – Strong desire or compulsion to take the substance
little anxiety and progressing slowly through more difficult – Difficulty in controlling substance taking behaviour
one. I f relaxation technique is also carried out w ith
this, it is know n as systemic densitization.

( b) Flooding -
Exposure is intensive from the start i.e. the patient is
exposed to the situation which provokes maximum anxiety
and is allowed to remain in there, until the anxiety has
diminished. The process is then repeated with sub maximal
stimuli and so on.

( c) Exposure in everyday pract ice -


• Most of the ex posure t / t is car ried out in a w ay 187. Disulfiram:
intermediate in speed and intensit y between those A. Inhibits alcohol dehydrogenase
used in desensitization and flooding and if possible in B. Inhibits aldehyde dehydrogenase
practice rather than in imagination. C. Both
D. None
Covert Sensitization - C
• It is an aversion therapy .........( AI PGMEE - 199 9)
• It is used for the t/t of conditions which are pleasant but
underirable e.g. Alcohol dependance. Tranvest ism ,
homosexuality and other sexual deviations.
• In aversion therapy pairing of the pleasant st imulus
with an unpleasant response ( e.g. brief elect rical
st i m u l us) is done so pleasant stimulus becomes
unpleasant by association.
• W hen t he unpleasant st im ulus is pr oduced by
fant asy instead of electric st imulus or drugs t he
procedure is know n as covert sensitization.
• Disulfiram inhibits the enzyme Aldehyde dehydrogenase .
TOPI C 11 : ALCOHOLI SM • Inhibition of aldehyde dehydrogenase with disulfiram is
irreversible ; synthesis of fresh enzyme is required for
18 6. By w hich, Alcohol dependence is best indicated: return of activity
A. Blackout
B. Early morning drinking However, it inhibits a number of other enzymes as well,
C. Withdrawal symptoms including:
D. Physical complications (i) alcohol dehydrogenase
C (ii) dopamine b hydroxylase.
.........( AI I MS PGMEE - MAY 199 5) (iii) several cytochrome p450 iso-
enzymes.
• Dependence syndrome : Thus it prolongs t 1/ 2 of many drugs.
“It is a Cluster of physiological behavioural and cognitive Th ou gh d isul f i r a m in hi bi t s b ot h t h ese en zy m e s
phenomena in which the use of substance takes much mentioned, its mechanism of action is because of inhibition
higher priority for a given individual than other behaviours of enzyme Aldehyde dehydrogenase only.
that once had greater value.”
18 8. Morbid Jealousy is seen w ith
A. Alcohol
B. Opium
C. Cannabis
D. Amphetamine
A
.........(AIPGMEE - 1994), PGI - JUNE 1997
Substance Symptom
Cocaine Magnus symptom (cocaine bues)
it is due to tactile hallucination
Cannabis Run Amok
Phenyclidine (Angel Dust) Dissociate anaesthesia
Diagnosis of dependence to any substance is made Alcohol Morbid jealousy
only if 3 of the following present
Me Ewan's sign – constricted
– Withdraw al Symptoms
– Progressive neglect of alternative pleasure Pupil on painful stumili dialates

HELP LI NE NO. 9 39 1 56 7 70 7
TOPI C 12: DELI RI UM TREMENS PSYCHI ATRY 45

18 9. Feat ures of alcohol w ithdraw al are all except: 19 3. W ernicke’s encephalopathy involves w hich part of
A. Hypersomnolence CNS :
B. Epileptic seizure A. Mammillary body
C. Restlessness B. Thalamus
D. Hallucination C. Frontal lobe
A D. Arcuaie fasciculus
.........( AI PGMEE - 199 8) A
.........( PGI - June -200 0)
• Mild withdrawl symptoms include : nausea, vomiting,
weakness, irritability, anxiety and insomnia, mild tremors • Wernicke’s encephalopathy involves mammillary body of
• Severe withdrawal syndrome is characterised by one of the CNS. It can also involve the 3rd and 4th ventricles
the following 3 disturbances : adjacent to mammillary bodies.
1. Deler ium t remens : Characterized by : clouding of
consciousness hallucinations 19 4. Psychiat ric complications of alcohol dependence:
2. Alcoholic seizures : Generalized tonic – clonic seizures: A. Anxiety
B. Suicide
Autonomic features : tachycardia , pupi dilation, fever, C. Depression
sweating, hypertension, insomnia. D. Schizophrenia
Insomnia psychomotor agitation and ataxia E. Mania
3. Alcoholic hallucinat ions : Characterised by presence of A, B & C
hallucinations usually auditory .........( PGI - June -200 1)

Hyersomnia may be seen as a feature in cocaine w ithdraw l. Ph y ch i a t r i c d i sor d e r a ssoci a t e d w i t h a l coh ol


dependance are :
• Alcohol induced mood disorder — depression, sadness.
• Suicide
• Severe anxiety disorder.
• Hallucinations and /or paranoid delusion — alcohol induced
psychotic disorder.
• Personality disorder (Comorbidity)

TOPI C 12 : DELI RI UM TREMENS


19 0. I n alcohol w ithdraw al, drug of choice is:
19 5. A 30 -year old male w it h hist ory of alcohol abuse
A. TFP
for 15 years is brought to the hospit al emergency
B. Chlormethazole
w it h com plaint s of t earf ulness, m is- r ecognit ion,
C. Chlordiazepoxide
talking t o self, aggressive behavior, tremulousness
D. Buspirone
and seeing snakes and reptiles that are not visible
C
to ot hers around him. There is hist ory of drinking
.........(PGI - JUNE 1997), AIPGMEE - 1995
alcohol t w o days prior to t he onset of the present
complaint s. He is most likely suffering from:
• In alcohol withdrawal syndrome, drug of choice are
A. Delirium tremens
benzodiazepines. Chiordiazepoxide (8 200 mg/ day in
B. Alcoholic hallucinosis
divided doses) & diazepam (40-80 mg/ day irt divided
C. Schizophrenia
doses) are the nv frequently used benzodiazepines.
D. Seizure disorder
• Buspirone has no role in alcohol withdrawal.
A
.........( AI I MS PGMEE NOV - 200 3)
1 9 2 . D r u g s u se d i n a l coh ol w i t h d r a w a l d u r i n g
maintenance phase:
It is a case of Delirium tremens
A. Naltrexone
It is the most severe alcohol w it hdraw al syndrome
B. Naloxone
It usually occurs w ithin 2- 4 days of complete or significant
C. Acamprosate
abstinence.
D. Disulfiram
A, C & D
Symptoms are
.........( PGI - JUNE 200 6)
• Clouding of consciousness
• Poor attention span
• Rehabilitation of agents for alcoholics :
• Visual hallucinations which are often vivid and frightening
- Acamprosate
• Tactile hallucinations and auditory hallucinations
- Naltrexone
• Marked autonomic disturbances
- SSRIs (fluoexetine)
- Disulfiram
Tachycardia Fever Sweating Hypertension Pupillary dilatation
• Alcohol deterrent agents (alcohol sensitizing drugs) :
• Psychomotor agitation and ataxia
- Disulfiram
• Insomnia
- Citrated calcium carbinide
• Dehydration
- Metronidazole
- Methyltetra zolethiol
The course is short with recovery occurring within 3-7 days
Animal charcoal, fungus coprius atramentarius.

HELP LI NE NO. 9 39 1 56 7 70 7
TOPI C 12: DELI RI UM TREMENS PSYCHI ATRY 46

Alcoholic hallucinosis The course is short, with recovery within 3-7 days.
• This can be ruled out easily because it is characterized by
the presence of auditory hallucinat ions and the other This is an acute organic brain syndrome with the characterist ic
usual features of delirium tremens are absent. features of :
1. Clouding of consciousness w ith disorientation in time
19 6. Most common symptom of alcohol w ithdraw al is: and place
A. Bodyache 2. Poor attention span and distractibility
B. Tremor 3. Visual ( and also audit ory) hallucinat ions and illusions,
C. Diarrhoea tactile hallucinations of insects crawling over body may
D. Rhinorrhea occur
B 4. Marked autonomic disturbance w ith tachycardia , fever,
.........( AI PGMEE - 200 7) sweating, hypertension and pupillary dilation
5. Psychomot or agit ation and ataxia
Tremulousness (shake, tremor or Jitter) is the classical & most 6. Insomnia, with a reversal of sleep-w ake pattern
common sign of alcohol withdraw ft. 7. Dehydration with electrolyte imbalance
Hangover ( nex t m or ning ) is the most comomon
symptom of alcohol withdrawl syndrome.

197 . A 40-year old man presents to casualty w ith hist ory


of regular and heavy use of alcohol for ten years and
morning drinking for one year. The last alcohol intake
was three days back. There is no history of head injury or
seizures. On examinat ion, there is no ict erus, sign of
hepatic encephalopat hy or focal neurological sign.
The patient had coarse tremors, visual hallucinat ions
a nd ha d d isor i en t at i on t o t i m e . W hi ch of t h e
follow ing is the best medicine to be prescribed for
such a patient?
A. Diazepam
B. Haloperidol 2 0 0 . Delirium t remens is charact erized by confusion
C. Imipramine associated w it h:
D. Naltrexone A. Autonomic hyperactivity and tremors
A B. Features of intoxication due to alcohol
.........( AI PGMEE - 200 4) C. Sixth nerve palsy
D. Korsakoff psychosis
Alcohol w ithdraw l Syndrome A
1. Hangover is most common presentation. .........( AI PGMEE - 200 3)
__
2 . Alcohol hallucinosis presents with usually auditory 20 1. An alcoholic is brought to the Emergency OPD w ith
hallucinations & clear consciousness the complaint or irrelevant talking. He had stopped
3. Alcoholic seizures / rumfits presents with generalized using alcohol three days back. On examination, he is
tonic clonic seizure, drug of choice being benzodiazepines. found to be disoriented to time, place and person.
4. Delirium tremens presents with acute onset clouding He also has visual illusions and hallucinations. There
of consciousness, disorientation, hallucination (M.C. is no history of head injury. The most likely diagnosis
vivid & frightening visual hallucination> auditory> tactile), is:
disturbed sleep w ith psychomotor a g i t a t i on A. Demetia praecox
( aggressiveness, tremors) & autonomic disturbance B. Delirium tremens
(Increased P.R/B.P/ sweating/ temperature) C. Schizophrenia
Drug of choice is chlordizepoxide > diazepam D. Korsakoff’s psychosis
B
19 8. An alcoholic is brought to the casualty, 3 days aft er .........( AI PGMEE - 200 3)
he quit alcohol, w it h t he complaint of irrelevant
t alk ing. On exam inat ion, he is found t o be dis The patient is presenting with a classical history of abstinance
oriented t o time, place and person. He also has visual from drinking.
illusions and hallucinations. ‘There is no hist ory of The onset of symptoms are with in 2-4 days of abstinence.
head injury. The most probable diagnosis is: The patient is having visual hallucinations and is disoriented
A. Dementia praecox both of which are characteristic features of Delirium
B. Delirium tremens tremens.
C. Schizophrenia
D. Korsakoff psychosis 20 2. True about delirium tremens :
B A. Clouding of consciousness
.........( AI PGMEE - 200 2) B. Coarse tremor
C. Chronic delirious behavior
Delirium tremens: D. Hallucination
It is the most severe alcohol w ithdraw l syndrome E. Autonomic dysfunction
It occurs usually within 2-4 days of complete or significant All
abstinence from heavy alcohol drinking. ..........( PGI - JUNE 200 5)

HELP LI NE NO. 9 39 1 56 7 70 7
TOPI C 13: PERSONALI TY DI SORDER PSYCHI ATRY 47

20 3. All are true about Delirium t remens except Type B Symptoms (Adrenergic Hyperactivity)
A. Severe depression 5 Is nausea of vomiting present?
B. Hallucination 6 Is a tremor visible with or without arms extended?
C. Extreme anxiety 7 Is sweat visible on palms or forehead?
D. Delusion 8 Is the systolic blood pressure greater than 140 mm Hg?
A 9 Is the diastolic blood pressure greater than 90 mm Hg?
.........( AI I MS PGMEE - JUNE 199 8) 10 Is the heart rate greater than 100 beats per minute?
11 Are there extra or skipped beats on apical pulse?
• Delirium tremens is most severe Alcohol w ithdraw al
syndrome with characteristic feature of – • Clonidine and BETA – block ers have been used
1. Clouding of concsiousness successfully in the treatment of specific symptoms
Disorientation related t o increased catecholamine output seen in
2. Hallucinat ion AW S
3. Agitation and Ataxia
4. I nsomnia Type C symptoms (Delirium)
5. Autonomic disturbance Does the patient:
6. Dehydration 12. Respond inappropriately to questions?
13. Report hearing noises that are not there?
Diagnostic Criteria for Alcohol Withdrawal 14. Report hearing noises that are not there?
A Cessation of (or reduction in) alcohol use that has been 15. Not know their name?
heavy and prolonged. 16. Not know where they are?
B Two (or more) of the following developing within several 17. Not know how long they have been hospitalized?
hours to a criterion A: 18. Not know the year?
1 autonomic hyperactivity (e.g., sweating or pulse rate 19. Not know the month?
greater than 100 beats per minute) 20. Not know the day of the week?
2 Increased hand tremor
3 Insomnia • Proposed mechanisms for t his sympt om clust er
4 Nausea or vomiting include increased dopamine release and enhanced
5. Transient visual, tactile, or auditory hallucinations or illusions dopamine recept or act ivit y, and NMDA recept or
6. Psychomotor agitation hypersensit ivit y.
7. Anxiety • Delir ium in AW S m ay be further exacerbated by
8. Grand mal seizures excessive doses of benzodiazepines for t reat ment of
C The symptoms in criterion B cause clinically significant Type A symptoms.
distress or impairment in social, occupational, or other
important areas of functioning. • Neuroleptic drugs, such as haloperidol, are moderately
D The symptoms are not due to a general medical condition effective in controlling this symptom cluster, but Type C
and re not better accounted for by another mental symptoms require 2- 10 days to resolve regardless
disorder. of treatment

Symptoms of Alcohol Withdrawal Syndrome TOPI C 13 : PERSONALI TY DI SORDER


Symptoms Time of appearance
after cessation of 20 4. A young lady w as admitted after taking overdose
alcohol use of diazepam after broken affair, she has hist ory of
Minor withdrawal symptoms: insomnia, 6 to 12 hours slitting her w rist previously. Most likely diagnosis:
tremulousness, mild anxiety , gastrointestinal A. Narcisstic personality disorder
upset, headache, diaphoresis, palpitations, B. Dependent personality disorder
anorexia
C. Borderline personality disorder
Alcoholic hallucinosis : visual, auditory , or tactile 12 to 24 hours
D. Histrionic personality disorder
hallucinations
C
Withdrawal seizures : generalized tonic-clonic 24 to 48 hours†
seizures .........( AI I MS PGMEE JUNE - 200 0)
Alcohol withdrawal delirium (delirium tremens): 48 to 72 hours‡
hallucinations (predominately visual),
disorientation, tachycardia, hypertension, low-
grade fever, agitation, diaphoresis

• In alcohol w it hdraw al syndr om e psychom ot or


act ivit y is increased so there will be Anxiety (not
depression), Insomnia (not drowsiness or hypersomnia).

Type A Symptoms (CNS Excitation) Does patient appear:


1 Anxious or nervous?
2 Restless? • Following points favour the diagnosis of Borderline
3 Bothered by bright light? personality disorder
4 Bothered by loud sounds? • Impulsive behaviour
• Unstable emotional responses
• Benzodiazepines are the cornerst one of treatment • Suicidal attempts
for this cluster A of symptoms.

HELP LI NE NO. 9 39 1 56 7 70 7
TOPI C 13: PERSONALI TY DI SORDER PSYCHI ATRY 48

• Dependent personalit y disorder • lacks empathy


• It denotes a chronic excessive dependence on ot hers. • is often envious or believes others are envious of him or
Such pts frequently defer decision making t o others. her
– They tolerate mistreatment, place other needs before • Arrogant affect.
own and have difficulty being assertive. pts also • Example-
demonstrate low self esteem, insecurity and longing to “A 48 yr old man is admitted to the ICCU following acute
be in a relationship. MI , a psychiatry consultant is requested when nurses
A mnemonic that can be used to remember the criteria for report that his offensive behavior is disrupting t he
dependent personality disorder is DEPENDENT. st aff and other pat ient s.
D – Difficulty making everyday decisions When questioned, the patient is dismissive and prefers
E – Excessive lengths to obtain nurturance and support from instead t o discuss his ‘beaut iful girlfriends’ new
others sports car, w idespread influence .
P – Preoccupied with fears of being left to take care of self He denies having had a heart attack and claims that it
E – Exaggerated fears of being unable to care for himself or was just ‘minor heartburn”.
herself
N – Needs others to assume responsibility for his or her life
D – Difficulty expressing disagreement with others
E – End of a close relationship is the beginning of another
relationship
N – Noticeable difficulties in initiating projects or doing things
on his or her own
T – “Take care of me” is his or her motto

• Example:
“A 44 year old female, a housewife complains of sorrow
due to break up of her marriage. She states that she
feels insecure w hen left on her ow n and has great
difficulty asserting herself . She adds that she considers
herself a ‘follower’ who has left all decision making to her
husband. She describes her husband as an intense and
domineering man. She has met through her husband.”

Narcisstic personality disorder


At least five of the following are necessary for a diagnosis
(as with many DSM diagnoses, they must form a pervasive
pattern; for example, a person who shows these criteria
only in one or two relationships or situations would not
properly be diagnosed with NPD):
• has a grandiose sense of self-importance is preoccupied
with fantasies of unlimited success, power, brilliance,
beauty, or ideal love
• believes that he or she is “special” and unique and can
only be understood by other special people
• requires excessive admiration
• strong sense of entitlement
• takes advantage of others to achieve his or her own ends

HELP LI NE NO. 9 39 1 56 7 70 7
TOPI C 13: PERSONALI TY DI SORDER PSYCHI ATRY 49

• The main dif ference b/ w avoidant per sonalit y


disorder and schizoid personalit y disorder is that
schizoid individuals are happy being alone, w hereas
avoidant individuals are distressed by their loneliness

20 5. Characteristics finding of schizoid personality is:


A. Emotional coldness
B. Conversion disorder
C. Not concerned with disease or anything
D. Check details of all things
A The DSM-IV-TR, a widely used manual for diagnosing mental
.........( AI I MS PGMEE NOV - 199 9) disorders, defines schizoid personality disorder as:
A pervasive pattern of detachment from social relationships
• Char act er s t ics of Schizoid t ype of personalit y and a restricted range of expression of emotions in
disorder interpersonal settings, beginning by early adulthood and
• can be best understood with help of an example present in a variety of contexts, as indicated by four (or
• A 40 yr old scientist lives alone, has no contact with his more) of the following:
family, does not have any friends. He has never engaged
in any social act ivities. • neither desires nor enjoys close relationships, including
• He prefers to w ork alone at home and cannot name being part of a family
any hobbies or activities t hat he finds enjoyable . • almost always chooses solitary activities
He has limited capacity to express either warm tender • has litt le, if any, interest in having sexual experiences
feelings or anger (emotionally cold) with another person
• takes pleasure in few, if any, activities
• “Voluntary social w ithdraw al w ithout psychosis” • lacks close friends or confidants other than first-degree
• A closely relat ed t ype of personalit y disorder is relatives
avoidant personality disorder. • appears indifferent to the praise or criticism of others
• shows emotional coldness, detachment, or flattened
affect ivity

Avoidant Personality disorder


A 23 yr old male complains of having too few friends and
an unreasonable fear of nerve experiences, he tells that
his shyness has frequently participat ing in social
activities.

HELP LI NE NO. 9 39 1 56 7 70 7
TOPI C 13: PERSONALI TY DI SORDER PSYCHI ATRY 50

• Borderline Personality Disorder (DSM-IV Personality


Disorders (BPD) is defined as a mental illness primarily
characterized by emotional dysregulation, extreme “black
and white” thinking, or “splitting”, and chaotic relationships.

• The general profile of the disorder also typically includes a


pervasive instability in (1) mood, (2) interpersonal
relationships, (3) self-image, (4) identity, and (5) behavior,
as well as a disturbance in the individual’s sense of self
this may also include impersonal manipulation.
• In extreme cases, this disturbance in t he sense of self
can lead to periods of dissociation

• A commonly used mnemonic to remember some features


of borderline personality disorder is PRAI SE:

• P - Paranoid ideas
• R - Relationship instability
• A - Angry outbursts, affective instability, abandonment
fears
• I - Impulsive behavior, identity disturbance
• S - Suicidal behavior
• E - Emptiness

206 . A female presents w ith t he history of slashed w rists


and att empt ed suicide, now present w it h similar
history. The diagnosis is: The criteria are:
A. Borderline personality disorder • Frantic efforts to avoid real or imagined abandonment.
B. OCD [Not including suicidal or self-mutilating behavior covered
C. Conversion Reaction in Criterion 5]
D. Histrionic personality • A pattern of unstable and intense i nterpersonal
A relationships characterized by alternating between
.........( AI I MS PGMEE NOV - 200 1) extremes of idealization and devaluation.
• I dentity disturbance: markedly and persistently unstable
symptomatic profile of the pt. self-image or sense of self.
is in accordance with Borderline personality disorder.
Characteristics of Borderline personalit y disorder is • I mpulsivity in at least tw o areas that are potentially
- self- damaging (e.g., spending, promiscuous sex, eating
(1) Unstable mood & behaviour disorders, substance abuse, reckless driving, binge eating).
(2) Suicide attempts – [Again, not including suicidal or self-mutilating behavior
(3) Boredom covered in Criterion 5]
(4) Feeling of emptiness & loneliness • Recurrent suicidal behavior , gestures, threats, or self-
(5) Impulsiveness mutilating behavior.

HELP LI NE NO. 9 39 1 56 7 70 7
TOPI C 13: PERSONALI TY DI SORDER PSYCHI ATRY 51

• Affect ive instabilit y due to a marked reactivity of mood • impulsively attempts suicide without any secondary motive
(e.g., intense episodic dysphoria, irritability, or anxiety to gain sympathy.
usually lasting a few hours and only rarely more than a
few days). Example of Borderline personality disorder
• Chronic feelings of empt iness. “During a routine physical exam
• I nappropriate, intense anger or difficulty controlling =
anger (e.g., frequent displays of temper, constant anger, a young female patient tells her physician that she has fallen
recurrent physical fights). in love with him
• Transient, stress- related paranoid ideat ion or severe =
dissociative symptoms. when he recommends that she see another physician
=
Diagnosis can be confused with Histrionic personality disorder She threatens to commit suicide.”
-
Characteristic of Histrionic personality disorder 20 7. A 16 -year old girl w as brought to the psychiatric
(1) Dramatic, extrovert & emotional emergency aft er she slashed her w rists in an at tempt
(2) Exihibit sexually provocative behaviour to commit suicide. On enquiry her fat her revealed
(3) unable to maintain intimate relationships that she had made several such attempts of w rist
although they often overstate the closeness of their slashing in the past, mostly in response to trivial fights
friendship in her house. Further she had marked fluctuations in
(4) Fundamentally insecure her mood w it h a per vasive pat t er n of unst able
and their theatrics are generally efforts to obtain love i nt e r pe r son a l r e l at i onsh i p. Th e m ost p r ob a bl e
support and reassurance, diagnosis is :
A. Borderline personality disorder.
Diagnostic criteria (DSM-IV-TR) B. Major depression
• is uncomfortable in situations in which he or she is not C. Histrionic personality disorder.
the center of attention D. Adjustment disorder
• inter action with others is often char acterized by A
inappropriate sexually seductive or provocative behavior ..........( AI I MS PGMEE NOV - 200 2)
• displays rapidly shifting and shallow expression of emotions
• consistently uses physical appearance to draw attention 20 8. Antisocial personalit y I s seen w ith :
to self A. Drug abuse
B. Paranoid schizophrenia
• has a style of speech that is excessively impressionistic C. OCN
and lacking in detail D. None
• shows self-dramatization, theatricality, and exaggerated A
expression of emotion .........( PGI - 1 999 - Dec)
• is suggestible, i.e., easily influenced by others or
circumstances • Antisocial personality disorder is commonly associated with
• considers relationships to be more intimate than they drug abuse.
actually are.
20 9. True about personalit y disorder : ( PD) :
mnemonic that can be used to remember the criteria for A. Typically onset at early childhood & adolescent
histrionic personality disorder is PRAISE ME B. Matured around age 30 - 40 yrs
• P - provocative (or seductive) behavior C. Egodystonic
• R - relationships, considered more intimate than they are D. Dramatic, emotional and erratic behaviour in paranoid PD
• A - attention, must be at center of E. Pervasive and maladaptive behaviour
• I - influenced easily A, B & E
.........( PGI - JUNE 200 3)
• S - speech (style) - wants to impress, lacks detail
• E - emotional lability, shallowness • Followings are the diagnostic criteria (according to ICD-
• M - make-up - physical appearance used to draw attention 10) in a person not having organic disease) for personality
to self disorder:
• E - exaggerated emotions - theatrical - Long standing dysharmonious attitudes and behaviour
involving several areas of functioning.
• can be better explained with an example - Pervasive and maladaptive behaviour.
• “A 27 yer old, charming, scantily clad w omen - Onset alw ays during childhood or adolescence.
– complains of suicidal feelings - Cont inuat ion t o adulthood.
– She explains that she is annoyed by a co-worker who has - Considerable personality distress (sometimes appear late
become centre of attraction at the office in the course of the disorder).
has a history of multiple suicide attempts - And usually, not always significant problems in work and
after which she claims to have received sympathy from social behaviour.
a multitude of ‘close friends’. • Personality disorder is alloplastic (can adopt to and alter
the exernal environment) and ego syntonic (acceptable
While a Histrionic personalit y disorder attempts to ego).
– suicide to gain sympathy and attraction • Personality disorders are less obvious in later years of life
a border line personality disorder pt. (>40 yrs.).

HELP LI NE NO. 9 39 1 56 7 70 7
TOPI C 14: HYPOCHONDRI ASI S PSYCHI ATRY 52

• Although they may exhibit some or all of these


Cluster A Cluster B Cluster C
characteristic it does not mean that people with type A
Odd and eccentric Dramatic, emotional Anxious and fearful
personality are incapable of showing love, affection or
- Paranoid PD and erratic - A voidant other types of non pessimistic behaviour.
- Schizoid PD - Antisocial - Dependent • Those who do not seek treatment are described as stress
- Schizotypal PD - Histrionic - Obsessive junkies and often display some of t he following
- Narcissistic
compulsive
characteristics
- Borderline
(1)An intrinsic insecurity or insufficient level of self esteem
which is considered to be the root cause of the syndromes.
This is believed to be covert and therefore less observable.
21 0. A person having the habit of repeated self- inflict ed (2) Time urgency and impatience which causes irritation and
injuries, w hat is t he type of personality exasperation.
A. Borderline personality disorder (3) Free floating hostility which can be triggerred even over
B. Schizoid personality disorder little incidents.
C. Histrionic personality disorder
D. Narcissistic personality disorder • The type B personality in contrast is relaxed and easy
E. Depressive disorder going.
A • There is also a type AB mixed profile for people who
.........( PGI - JUNE 200 4) cannot be clearly categorized and have a combination of
21 1. Rorschach test measures :
both types of personality.
A. Intelligence
B. Creativity
C. Personality
D. Neuraticism
C
.........( PGI - June -199 9)

• Rorschach inkblot test measures PERSONALITY.


• Other tests for personality:
— Thematic apperception test (TAT)
— Draw - a person test (DAPT)
— Sentence completion test.
• Tests for’INTELLIGENCE’
• Wechsler adult intelligence scale (WAIS) TOPI C 14 : HYPOCHONDRI ASI S
• Standford - Binet test
• Bhatia’s battery of intelligence test. 21 3. Hypochondriasis is
A. Fear of fatal disease
21 2. Type ‘A’ personality is characterized by all except, B. Too much concern of own health and misinterpretation of
A. Hostility normal body sensation
B. Time pressure C. Marked mental and physical fatigue
C. Competitiveness D. Repeated hospitalization with depressive symptoms
D. Mood fluctuations B
D .........( AI I MS PGMEE - MAY - 199 4)
.........( AI I MS PGMEE - NOV 200 7)
Type A personality disorder also known as type A behaviour 214. One of the follow ing usually differentiates hysterical
pattern is a set of characteristic that includes being :- symptoms from hypochondriacal symptoms:
• Impatient A. S ymptoms do not normally reflect understandable
• Excessively time conscious physiological or pathological mechanism.
• Insecure about one’s status B. Physcial symptoms are prominent which are not explained
• Highly competitive by organic factors.
• Hostile and aggressive
C. Personality traits are significant.
• Incapable of relaxation
D. Symptoms run a chronic course
• Type A indivi duals are often h i g h l y a ch i e v i n g
A
w orkaholics w ho do mult itask , drive themselves
..........( AI PGMEE - 200 5)
w ith deadlines and are unhappy about the smallest
of delays.
Symptoms in conversion disorders ( Hysteria) cannot
be explained by any physical disorder or know n
pathophysiological mechanisms

Conversion disorder (Hysteria)


Characterized by alteration of physical functioning that
suggests physical disorder but inst ead is actually an
expression of psychological conflict or need .
‘The symptom is not under voluntary control and cannot
be explained by any physical disorder or know n
pathophysiological mechanism’

HELP LI NE NO. 9 39 1 56 7 70 7
TOPI C 14: HYPOCHONDRI ASI S PSYCHI ATRY 53

• Case presentation of Hypochondriosis


• A student on a neurology placement reported interpreting
minor aches and twinges as being early symptoms of
multiple sclerosis. She woke up every morning for two
weeks convinced that she would be paralysed by an acute
attack. This health anxiety interfered with study and
caused considerable distress. I t resolved w hen she
finished the at tachment , but it recurred in other forms
Hypochondriasis during later courses.
characterized by preoccupation with fear of having a • A 48 year old woman had 77 admissions, 11 operations,
serious disease based on person’s misterpretation of bodily and 996 investigations in 25 years. She has been seen in
symptoms. 10 hospital specialties but, other than her psychiatric
The symptom is hypochodriasis is understandable and diagnosis, she has not received a diagnosis that explains
is based on normal physiological sensations or actual her symptoms. She has suffered from a number of serious
benign symptoms or signs. iatrogenic complications, including: opiate dependence;
It is only misinterpreted and exaggerated persists opi at e i nd uce d r e sp i r a t or y ar r e st ; Cu sh in g’s
despite medical reassurance. syndrome after a trial of steroids; and a pulmonary
embolism aft er a period of immobilit y.
• The interventions and investigations used repeatedly led
to further deterioration in her condition. The total financial
cost has been about Rs 8 Lakhs . After psychiatric
intervention her healthcare use decreased, graphically
shown by costs dropping from Rs 40000 to Rs 4000 a
year.

2 1 6. A 4 5 yrs old male came t o t he psychiatric OPD


com planing of cont inuous, dull, non- progressive
headache for the last 8 years. The pat ient has seen
numerous neurologists in teh belief t hat he has a
brain tumor even though all his invest igat ions have
been normal. The patient insist ed t hat he had a brain
tumor and requested yet another w orkup. Psychiatric
2 15 . A patient is alw ays preoccupied w it h feeling of e v a l u a t i on r e v e a l s d i se a se con v i ct i on i n t h e
illness. The Diagnosis is: backgr ound of nor m al invest igat ions. The m ost
A. Hypochondriasis probable diagnosis is:
B. Somatisation disorder : A. Hypochondriasis
C. Conversion disorder B. Somatization disorer
D. Obscession C. Somatoform pain disorder
A D. Conversion disorder
.........( AI PGMEE - 200 0) A
.........( AI PGMEE - 200 2)
Hypochondriasis is defined as persistent preoccupation with
fear or belief of having one or more serious diseases, based Patients with Multiple Somatic Complaints that cannot be
on person’s ow n int erpret at ion of normal body explained may have
function or minor physical abnormality.
: The fear or belief in not a delusion . Patient may agree,
Hypochondriasis Somaization Disorder
regarding the possibility of his exaggeration t he Essential feature is belief of serious medical • Onset is usually before 30 yrs.
graveness of sit uation at t hat time. illness that persists despite reassurance • Disorder is persistent
and appropriate medical evaluations • Multiple organ symptomatology i.e. At least 4 pain,
2 gastrointestinal, one sexual and one
• Pre-occupation with medical terms and syndromes is pseudoneurological symptom.

common.

• Hypochondriasis –
Patient may agree, regarding the possibility of his Malingering Factitious Illness
exaggerat ion t he graveness of sit uat ion at t hat Mostly involve voluntary motor or sensory • Patient voluntarily produces Symptoms
functions
time.

HELP LI NE NO. 9 39 1 56 7 70 7
TOPI C 15: OPI OI D PSYCHI ATRY 54

• Malingering is defined as “the intentional production of (1) Lacrymation


false or grossly exaggerated physical or psychological (2) Sweating
symptoms motivated by ext ernal incentives (3) Yaw ning
(4) Anxiety
(5) Fear
(6) Restlessness
(7) Goose flesh
(8) Mydriasis
(9) Tremor
(10) Insomnia
(11) Abdominal colic
(12) Diarrhoea
21 7. Hypochondriasis is (13) Dehydration
A.Normal pre occupation with abnormal body function (14) Risein B.P.
B.Abnormal pre occupation with abnormal body function (15) Palpitation
C.Normal pre occupation with normal body function (16) Rapid weight loss
D.Abnormal preoccupation with normal body function T/T → Consists of withdrawal of the opioid and substitution
D with oral metbadone followed by gradual withdrawal of
.........( AI PGMEE - 199 4) methadone

2 1 8 . A 4 1 - year old married female present ed w it h • Features of cannabis w it hdraw al -


headache for t he last 6 mont hs. She had several • Cannabis produces a very mild physical dependence with a
consultat ions. All her investigations w ere found to relatively mild withdrawal syndrome
be w it hin normal limits. She still insists that there is • It is characterized by -
somet hing w rong in her head and seeks anot her Fine tremors
consultat ion. The most likely diagnosis is: Irritability
A. Phobia Restlessness Nervousness Insomnia
B. Psychogenic headache Decreased appetite and craving
C. Hypochondriasis
D. Depression Features of cocaine w ithdraw al
C • Cocaine use produces a very mild physical but a very
.........(AIPGMEE - 2003), PGI - June -1998 strong psychic dependence.
Alcohol w ithdraw al
• The patient in question is persistently preoccupied about
her disease { headache) despite of all normal investigations It is characterized by marked autonomic disturbances with
and seeks another consultation as she still insists that there visual and auditory hallucination.
is something wrong in her head.
22 2. Nalt raxone is used in a case of opioid dependence
22 0. A 35 year old male, w ith pre-morbid anxious traits t o:
and heavy smoker, believes that he has been suffering A. Prevent respiratory depression
from ‘lung carcinoma’ for a year, No significant clinical B. To treat withdrawal symptom
finding is det ect ed on exam inat ion and relevant C. To prevent relapse
invest igat ions. He cont inues to st ick t o his belief D. To treat overdose of opioid
despit e evidence to the contrary. I n the process, he C
has spent a huge amount of money, time and energy .........( AI PGMEE - 200 7)
in get ting himself unduly investigated. He is most
likely suffering from- Naltraxone is a long acting opioid antagonist. It is used in
A. Carcinoma lung patients of opioid dependence for detoxification and
B. Delusional disorder withdr awal. It d iscour a ge s su b st a n ce se e k in g
C. Hypochondriacal disorder behaviour and thus prevents relapse. It may lead to
D. Malingering withdrawal symptoms® in addicts d/t blockage of opioid
C receptors.
.........( AI I MS PGMEE - NOV 200 4)
22 3. Acut e opioid w ithdraw al is charact erized by
TOPI C 15 : OPI OI D A. Rhinorrohea
B. Pilo erection
22 1. Yaw ning is a common feat ure of C. Miosis
A. Alcohol with drawal D. Insomnia
B. Cocaine with drawl E. Constipation
C. Cannabis with drawl A, B & D
D. Opioid with drawl .........( PGI - DEC 200 4)
D
.........( AI I MS PGMEE MAY - 200 3) Opioid withdrawal can be graded 0-4 : Grade 0 = craving and
anxiety
Features of opioid withdrawal have been a regular feature of Grade 1 = Yaw ning, lacrim at ion, r hinor rhea and
AIMS and All India examinations so try remember as many perspiration
as possible.

HELP LI NE NO. 9 39 1 56 7 70 7
TOPI C 15: OPI OI D PSYCHI ATRY 55

Grade 2 = Previous symptoms plus mydriasis, piloerection, 22 6. All are adulterant s of heroin, except:
anorexia, tremors, hot and cold flashes with generalized A. Chalk powder
aching. B. Quinine
Grade 2 = Previous symptoms plus mydriasis, piloerection, C. Charcoal
anorexia, tremors, hot and cold flashes with generalized D. Fructose
aching. C
Grade 3 & 4 = intensity of previous symptoms and signs plus .........( AI I MS PGMEE - MAY 200 5)
increased temperature, blood pressur e, pulse and
respiration rate and depth. • Following are important adulterants in heroin
Most severe = Vomiting, d i a r r h e a , weight loss, Quinine
haemoconcentration and spontaneous ejaculation or - Caffeine
orgasm commonly occurs. Paracetamol
- Theophylline
22 4. I n patients of substance- abuse, drugs used are : Noscapine
A. Naltrexone Scopolamine
B. Naloxone Phenacetin
C. Clonidine Procaine
D. Lithium Phenobarbitol
E. Disulfiram Diphenhydramine
A, C and E Methaqualone
.........( PGI - June -200 2) Lidocaine
Strychnine
• Naltrexone is a narcotic antagonist which when given to Nutmeg
opioid dependent individuals, causes withdrawal symptoms - Chalk
which is managed by clonidine. Starch (Fructose, Sucrose) Talcum powder
• Naltrexone also reduces alcohol craving - Powdered Milk
• Disulfiram is used in de-addition of alcohol. Flour
• Naloxone is opiod antagonist. • Adulter ants are mixed with heroin to increase its
• Lithium is used as mood stabiliser. pharmacological effect or to increase its weight.

22 5. W hich of the follow ing is not an opioid peptide? 2 2 7 . W hi ch of t he f oll ow i n g i s an al t er n at iv e t o


A. p-Endorphin met hadone for maint enance t reat ment of opiat e
B. Epinephrine dependence?
C. Leu5-enkephalins A. Diazepam
D. Met-enkephalin B. Chlordiazepoxide
B C. Buprenorphine
.........( AI I MS PGMEE - MAY 200 5) D. Dextropropoxyphene
C
• Our body produces some substances which have opium .........( AI I MS PGMEE - MAY 200 5)
like analgesic (pain killing) properties but chemically they
are peptides - hence they are called endogenous opioids. • Withdrawal from opoid dependence is usually managed
• These peptides are active in small amount. by substituting another opioid drug.
• Their actions are blocked by naloxone. - Methadone is the drug of choice.
• They bind with high affinity to the opioid receptors. • Methadone is a synthetic opioid, chemically dissimilar but
• They normally modulate pharmacologically very similar to morphine. It has an affinity
- Pain perception Mood for the mu receptors that is similar to that of morphine.
- Hedonic (pleasure related) and motor behaviour But it occupies it for longer time and its slow offset
- Emesis effect attenuates withdrawal symptoms.
• Methadone is also the preferred dr ug in opioid
(I) Endorphins- maintenance programme for addicts who decline to
Beta endorphin is the most important endorphin. withdrawn.
- It is derived from Pro-opiomelanocortin (POMC) Occupancy of opioid receptors by methadone reduces the
Beta endorphin is primarily a mu agonist desire for other opioids. Addicts feel less kicks/buzz/rush
from i.v. heroin or morphine because their opioid receptors
(II) Enkephalins - are already occupied by methadone.
These are derived from proenkephalin • There is risk of dependence with methadone but it is less
- Methionine enkephalin (Met - Enk) and leucine enkephalin severe than with morphine or heroin.
(Leu-Enk) are the most important. • Alternatives to methadone in maintenance therapy
(III) Dynorphins -
- These are derived from prodynorphin Buprenorphine -
• Recently a novel opioid peptide - Nociceptin / orphanin Buprenorphine is a [i opioid agonist and K
FQ (N/OFQ) has been isolated from mammalian brain. It is antagonist
localized in cortex, hippocampus and certain sensory sites. This drug has several advantages which are
It is believed to play a role in stress response, reinforcing - Low overdose danger
actions, learning and memory. - Easier detoxification than is seen with methadone
- Probable ceiling effect (higher doses does not increase
euphoria).

HELP LI NE NO. 9 39 1 56 7 70 7
TOPI C 16: PTSD PSYCHI ATRY 56

22 8. Nalt rexone is used in opioid addiction because • Symptoms resolves rapidly if stress is removed.
A. To treat withdrawl symptoms • If stress continues resolution of symptoms begin after, 1-2
B. To treat overdose of opioids days & minimal after 3 days.
C. Prevent relapse
D. Has addiction potential
C
.........( AI I MS PGMEE - MAY 200 7)

TOPI C 16: PTSD

2 2 9 . Three years back a w oman suffer ed during an


eart hquake and she w as successfully saved. Aft er
recovery she has night mares about the episode and
she also gets up in the night and feels t errified. The 23 1. W hich of the follow ing is NOT a clinical feat ure of
most probable diagnosis is : Post-Traumat ic Stress Disorder ( PTSD) ?
A. Major depression A. Flashbacks
B. Post-traumatic stress disorder B. Hyperarousal
C. Mania C. Hallucinations
D. Schizophrenia D. Emotional numbing
B C
.........( AI I MS PGMEE MAY - 200 2) .........( AI PGMEE - 200 8)

23 0. A lady w hile driving a car meet s w ith an accident. 23 2. A lady, w hile driving a car, meets in an accident.
She w as admitted in an I CU for 6 months. After being She w as admitted in an I CU for 6 months. After being
discharged, she oft en get s up in night and feels discharged, she oft en gets up in the night and feels
terrified and has fear to sit in car again. The diagnosis t er r ified and has f ear t o sit in a car again.The
is : Diagnosis is:
A. Panic Disorder A. Panic disorder
B. Phobia B. Phobia
C. Conversion disorder C. Conversion disorder
D. Post traumatic stress disorder D. Post Traumatic Stress Disorder,
D D
.........( AI I MS PGMEE NOV - 199 9) .........( AI PGMEE - 200 0)

• PTSD is type of reaction to stress & adjustment disorder PTSD arises as a delayed or protracted response to an
• Symptoms are not always immediate, there may be exceptionally stressful or catastrophic lie event, such as
latency. There is no clear temporal relation. It is disasters, rape, war or torture or serious accident.
characterized by It is characterised by recurrent and intrusive recollection
(a) Recollection of stressful events in dreams images or of the st ressfull event either in ‘flashbacks’ or in
thoughts. ‘dreams’.
(b) Sense of re experiencing stress
(c) Marked avoidance of situation that arouse recollection of There is as associated sense of re-experiencing of the stressful
stressful events event. There is marked avoidance of the event or situation
(d) Marked anxiety that arouse recollection of stressful event.
(e) Anhedonia (inability to exp. pleasure) Remember that symptoms of PTSD may develop after a
(f) Partial amnesia of stressful events period of lat ency, w ithin 6 months aft er t he stress
or may be delayed beyond this period.

23 3. A lady, w hile driving a car meets w ith an accidet.


She w as admit ted in an I CU for 6 moths. After being
disch- arged, she often gets up in night and feels
t errified .She is afraid t o sit in a car again. The
diagnosis is:
A. Panic disorder
B. Phobia
C. Conversion disorder
D. Post traumatic stress disorder
D
.........( AI PGMEE - 200 1)

23 4. Post traumatic stress syndrome is due t o:


A. Head injury
• Ot her stress reactions is
B. Minor stress
• Acute stress reaction →
C. Major life threatening events
• There is an Immediate & clear temporal relation b/w an
D. Vascular accidents
exceptional stress (death, accident, rape) and onset of
C
sympt.
.........( AI PGMEE - 199 8)

HELP LI NE NO. 9 39 1 56 7 70 7
TOPI C 17: AUTI STI C DI SORDER PSYCHI ATRY 57

PTSD is a delayed and protracted response to an exceptionally • Marked impairment in the use of multiple nonverbal
stressful or catastrophic life event or situation. behaviors such as eye-to-eye gaze, facial expression, body
It is characterised by recurrent and intrusive recollection of postures, and gestures to regulate social interaction
the stressful event either in flash backs or in dreams.

23 5. True about post traumat ic stress disorder :


A. Recall of traumatic events
B. Associated with major trauma like pelvic #
C. Treatment is ECT
D. Disturbed sleep
A, B & D • Failure to develop peer relationships appropriate to
.........( PGI - 2 000 - Dec) developmental level
• A lack of spontaneous seeking to share enjoyment,
• It is a delayed or protected response to on exceptionaly interests, or achievements with other people (eg, by a
stressful or catastrophic life event or situation in which lack of showing, bringing, or pointing out objects of
likely to cause pervasive distress in almost any person (e.g. interests)
disasters, war, rape, or torture and serious accidents) • Lack of social or emotional reciprocity
• It is characterised by recurrent and intrussive recollections
of the stressful event either in flash backs (images, (Group 2) Qualitative impairments in communication as
thoughts, or perceptions) and/or in dreams. manifested by at least 1 of the follow ing:
• Increased arousal, anxiety, anhedonia may occur. • Delay in, or total lack of, development of spoken language
• Drug treatment with anti-depresents and benzodiazepines (not accompanied by an attempt to compensate through
are useful. alternative modes of communication such as gestures or
mime)
23 6. Post traumatic stress- disorder I s associated w ith
all except :
A. Flash back
B. Severe traumatic injury
C. Re-experiencing stressful events
D. Anhedonia
E. It does’t develop after 6 months of stress
E
.........( PGI - June -200 0)

TOPI C 17 : AUTI STI C DI SORDER

23 7. A 6 yr old child has hist ory of Birth Asphyxia does • In individu als with a d e q u a t e sp e e ch , m a r k e d
not communicate w ell, has slow mental and physcial im pairm ent in t he abilit y t o init iat e or sust ain
gr ow t h, does not m ix w it h people, has lim it ed conversat ion w ith others
interests, gets w idely agitated if disturbed, diagnosis • Stereotyped and repetitive use of language or idiosyncratic
is: language
A. Hyperkinetic child • Lack of varied, spontaneous make-believe play or social
B. Autistic Disorder imitative play appropriate to developmental level
C. Attention deficit disorder
(Group 3) Re st r i ct ed r ep et it iv e an d st er eot y pe d
D. Schizophrenia
pat t er ns of behavior , interests, and activities, as
B
manifested by at least 1 of the following:
.........( AI I MS PGMEE NOV - 200 1)
• Encompassin g preoccupation with one or more
stereotyped and restricted patterns of interest that is
Symptoms of the child
abnormal either in intensity or in focus
H/o Birth Asphyxia
= strongly suggests
diagnosis of Autist ic Disorder .

Birth Asphyxia
a common cause of this disorder

• Three main clusters of behaviors define autism, as follows:


– (1) social abnormalities, especially a lack of social reciprocity;
• (2) language abnormalities, with deviant communication
features and limited development of language; and
• (3) rigid, stereotyped, repetitive patterns of unusual
behavior.

Group 1) Qualitative impairment in social interaction, as • Apparently inflexible adherence to specific, nonfunctional
manifested by at least 2 of the following: routines or rituals

HELP LI NE NO. 9 39 1 56 7 70 7
TOPI C 17: AUTI STI C DI SORDER PSYCHI ATRY 58

• Stereotyped and repetitive motor mannerisms (eg, hand - Intrusive sterotypes (Repetitive behaviour) together with
or finger flapping or twisting, or complex whole body inability’ to concentrate may prevent children from
movements) engaging in meaningful activity or social interaction
• Persistent preoccupation with parts of objects ( difficult y in making friends)

• Hyperkinetic child is characterized by


- Do nut engage in pretended play (which starts before
1. Poor attention span with distractibility
age of 2 in normal children)
2. Hyperactivity
3. Impulsivity • Mental retardation (about 75% of children with autism
are mentally retarde d) ‘
• Epilepsy develops in one fifth to one third of autistic
individuals
• EEG abno rmalities are noted in half of the patients.

23 9. Autism I s :
A. Neurodevelopmental disorder
B. Social and language commuiti^ation problem
C. Metabolic disease
D. Mainly due to hypothalamus damage
B
.........( PGI - 2 000 - Dec)

• Autism is a pe r v asiv e de ve lopm en t a l di sor d er


characterised by marked impairment in reciprocal social
and interpersonal interaction.

23 8. A 3 year old child presents w it h delayed speech


a n d p oor co n ce n t r a t i on . H e h a s d i f f i cu l t y i n
communication and is not making any friends. The
likely diagnosis is:
A. Autism
B. ADHD
C. Specific learning disability
D. Mental retardation
A
.........( AI PGMEE - 200 7) Ot her important feat ures :
— Lack of awareness of existence of others or feelings
Difficulty in concentration and communication, delayed — Absent social smile, lack of eye to eye contact
speech and problems in forming social relationships — Lack of attachment to parents and absence of separation
( not making any friends) in early childhood ( onset anxiety
of symptoms before the age of 3 years) suggests — No or abnormal social play, prefer solitary games
the diagnosis of Autism. — Lack of initiative, making friends
— Absence of fear in presence of danger.
Essentials of Diagnosis and Typical Features of Autism
• Onset in infancy or early childhood (before age of 3 years)
• Ab nor m a l la ngu ag e a nd sp ee ch de ve lop m e nt
( Abnormal communication)

- Inability to develop normal social skill (lack of eye contact


gestures and facial expression)
- Understand little or no language (therefore f ait t o
acquire speech )

- Deficient comprehension and communicative use of speech


and gesture.
24 0. Conduct disorder in a child manifest s w ith
• Behavioural peculiarities such as ritualized repetitive or A. Disregard for right of others
stereotyped behaviour and rigidity B. Doesn’t care for authority
• Difficult y in concentration C. Backward in studies
D. Decreased head circumference
• Severe deficit, in social responsiveness and interpersonal E. Steals things
skills A, B, C & E
.........( PGI - 2 001 - Dec)

HELP LI NE NO. 9 39 1 56 7 70 7
TOPI C 17: AUTI STI C DI SORDER PSYCHI ATRY 59

• I n cr e a se d se r ot on i n l e v e l , I n cr e a se d CSF
Homovanillic acid ( HVA) , Decreased CSF rat io of
HI AA t o HVA

• Abnormalities in plasma, platelet and urinary levels of


norepinephrine, epinephrine and 3- methoxy 4- hydroxyl
phenyl glycol.
Endogenous opioid.
- Other factors: Genetic, perinatal, and immunological.

Clinical features:
i) Physical characteristics - Handedness, intercurrent physical
• Characteristic clinical Features of conduct disorder :
illness.
- Frequent lying
- Stealing or robbery
- Running away from home and school. ii Behavioral characteristics - Qualitative impairments in social
- Physical violence like rape, fire-setting, assault or break-in, interaction, disturbances of communication and language,
use of weapons. stereotyped behavior, instability of mood and affect, hypo
• Basic psychopathology — basic rights of others violated — or hyperresponsive to sensing stimuli, hyperkinesis.
rule of society are not followed.
iii) Impaired intellectual functioning.
> D / D : D e a f n e ss, i m p a i r e d v i si on , ch i l d h ood
schizophrenia, repressive psychosis.
> Treat ment :
• Pharmacological agents : Haloperidol, Risperidone, SSRI,
clomipramine, antiepileptics, anxiolytics, p-blockers, and
stimulants,
• ii) Behavioral and educational treatment.

24 2. Autism I s:
24 1. I nfantile autisim is characterized by: A. Biological causation
A. Impaired vision B. Pervasive social and language communication problem
B. Impaired Neurobehavioral development C. Metabolic disease
C. Impaired folet level D. Mainly due to hypothalamus damage
D. A socioeconomic hazard E. Onset after 2-j- yrs. Usually
E. Parenting A And B
B& D .........( PGI - June -200 1)
.........( PGI - DEC 200 4)
• Autism is a pervasive developmental disorder. Presently
• Autistic disorder (sometimes called early infantile autism, the casue of infantile autism is predomiantly biological.
childhood autism or Kanner’s autism) is characterized by • Autism is marked impairement in reciprocal social and
marked abnormal development in social int eract ion interpersonal interaction.
and communicat ion and rest rict ed repert oire of • Onset typicaly occurs before the age of 2 years. In some
activities and interests. cases, onset may be later in childhood.
• Features of autistic disorder:
> Prevalence 2-5 / 10,000 children 243 . A girl w ith normal milestones spend her time seeing
> M : F: 3 : 5 her ow n hand, do not interact w ith ot hers, w hat is
> Prevalence increasing among low socioeconomic the diagnosis ?
groups. A. ADHD
B. Autism
Etiology and pathogenesis: C. Asperger’s syndrome
- It is a developmental behavioral disorder. D. Rett’s disorder
- Although the disorder was first considered to be B
psychosocial or psychodynamic origin, much evidence .........( AI I MS PGMEE - MAY 200 8)
suggests a biological substrate.
- Association with Biomedical conditions e.g. PKU, Tuberous Pervasive development disorder in a child.
sclerosis, fragile-X syndrome. • Prev asive de velopm en t al disor de r in clud es t he
- Abnormalities in EEG and MRI follow ing condit ion :-
- Increase in total brain volume (greatest involvement - Autism
in occipital, parietal and temporal lobes). - Rett’s disorder
- Asperger’s disorder
- Biochemical factors: - Childhood disintegrative disorder
- Pervasive disorders not otherwise specified

HELP LI NE NO. 9 39 1 56 7 70 7
TOPI C 17: AUTI STI C DI SORDER PSYCHI ATRY 60

Pervasive development disorders are charact erized by Asperger’s disorder


three symptom clusters- • The important point in differential diagnosis of these
i) I mpairment in social reciprocity marked by poor eye patients is that t hey have normal intelligence. These
contact, lack of interest in other people and failure to are the only pervasive development disorders in which
interact appropriately with others, the intelligence is unimpaired.

ii) Communicative impairment, it is characterized by either • These patients often have some except ional abilit y.
no language at all or by deviant speech with errors in
tone, prosody, pitch, grammar, or pragmatics, • They have little or no developmental language delay and
relatively normal cognitive development.
( iii) Restricted and repetitive behaviours include using
the same words and phrases repeatedly out of context, • In the question the lack of social int eract ion and
performing the same action in routine ways or insisting communicat ion along w ith stereot ypic behaviour
that others do so, or exhibiting other vocal or visual self (watching her own hard) suggests Autistic disorder.
stimulatory behaviours.Stereotype movements are seen.

(Stereotype movements are apparently purposeless


movements such as hand flapping, head rolling or body
rocking.)

Salient feat ure of each pervasive disorder: Aut istic


disorder
• It gives all the classical features of pervasive developmental
disorder.
• The patient presents with p r ob l e m s i n soci a l
interaction, communication and repetitive sterotype
behaviour.
• In most of these patients there is subnormal intelligence.

Example
• A 3 year old boy shows no interest in or connection to his • The patient does not give any symptoms of attention
parents, other adults or children. He does not speak deficit disorder i.e. there are no symptoms indicating lack
voluntarily and is fascinated w ith w atching rotating of concentration and hyperactivity.
obj ect s.
He screams fiercely when his environment is altered in • The patients of ADHD can also present with symptoms
any way such as when his mother tries to dress him. similar to pervasive developmental disorder but the main
complaint will always be that of at t ent ion deficit and
Rett’s disorder hyperact ivity.
• It is seen exclusively in females.
• The characteristic feature of these patients is that they
begin to lose their acquired skills.

Example
• After 4 months of normal development, an infant begins
to lose her acquired skills.
By 18 months of age, she shows little social interaction
with her parents, other adults or children and she uses
strange hand gestures.

HELP LI NE NO. 9 39 1 56 7 70 7

You might also like